Short clinical consultations Flashcards

1
Q

Management of migraine?

A

Acute phase: paracetamol / NSAID + antiemetic (1st line). Sumatriptan (oral / intranasal / subcut)

Longer term: remove triggers, exercise, sleep hygiene, decrease caffeine.

Prophylaxis = topiramate, propranolol

Avoid opiates / analgesia overuse

How well did you know this?
1
Not at all
2
3
4
5
Perfectly
2
Q

Typical migraine history?

A

Unilateral
Throbbing in nature
Light aversion / phonophobia
Aura / scintillating scotoma
Triggers: behaviours / foods
Onset: acute onset, can last days
Similar episodes in the past

How well did you know this?
1
Not at all
2
3
4
5
Perfectly
3
Q

Headache red flags to identify in the history

A

Vascular: maximal intensity 1-2 mins, ‘thunderclap’ headache (SAH)

Infective: pyrexial, confused, unwell, meningism, photophobia

SOL: insidious headache onset, worse on bending over / coughing / mornings

Posterior fossa syndrome: incoordination, truncal ataxia, nystagmus

Nausea / vomiting, visual changes, focal neurological deficit

Presentation with 1st headache over 55y

How well did you know this?
1
Not at all
2
3
4
5
Perfectly
4
Q

Approach to fundoscopic exam?

A

‘ROVM’

  1. Red reflex - obscured in cataracts, corneal scars, vitrous haemorrhage
  2. Optic disc - looking for cup, colour and contour. Normal cup : disc ratio is 0.3
  3. Vessels - start at disc, follow vessels to all 4 quadrants, ask patient to look up, down, left and right
  4. Macula - temporal to disc, use green light for foveal reflex
How well did you know this?
1
Not at all
2
3
4
5
Perfectly
5
Q

Fundoscopy features: glaucoma

A

Large cup : disc ratio (>0.5) indicating cupping of disc

Superior polar notching

Nasal displacement of central blood vessels

How well did you know this?
1
Not at all
2
3
4
5
Perfectly
6
Q

Fundoscopy features: papilloedema

A

Disc margins are obscured, swollen and hyperaemic

Retinal vessels are tortuous

NB must rule out SOL, signs of increased ICP

How well did you know this?
1
Not at all
2
3
4
5
Perfectly
7
Q

Fundoscopy features: optic atrophy with macular scarring

A

Optic disc pallor with cupping

Large area of macular scarring

ARMD = most common cause of macular scarring

How well did you know this?
1
Not at all
2
3
4
5
Perfectly
8
Q

Fundoscopy appearances: dry age-related macular degeneration

A

Atrophy of retinal pigment in central macula

Drusen in macular area

NB presence of haemorrhages + oedema in macular area suggests wet macular degeneration

How well did you know this?
1
Not at all
2
3
4
5
Perfectly
9
Q

Fundoscopy appearances: hypertensive retinopathy grades 1-4

A

Grade 1: arteriolar narrowing

Grade 2: AV nipping

Grade 3: exudates, haemorrhages, cotton wool spots

Grade 4: papilloedema

NB no microaneurysms! only present in diabetic retinopathy.

How well did you know this?
1
Not at all
2
3
4
5
Perfectly
10
Q

Fundoscopy appearances: retinitis pigmentosa

A

Multiple bony spicule, retinal black pigmentations scattered in the periphery of the retina

Associated history of poor night vision / blindness

Positive FH

Reduced visual fields / tunnel vision

How well did you know this?
1
Not at all
2
3
4
5
Perfectly
11
Q

Fundoscopy appearances: central retinal vein occlusion (CRVO)

A

‘Stormy sunset’: engorged retinal vein with retinal haemorrhages

How well did you know this?
1
Not at all
2
3
4
5
Perfectly
12
Q

Fundoscopy appearances: central retinal artery occlusion (CRAO)

A

Central ‘cherry red spot’ with surrounding pale retina (due to choroidal blood supply to macula remains intact)

Attenuation of arteries + veins

How well did you know this?
1
Not at all
2
3
4
5
Perfectly
13
Q

Fundoscopy appearances: branch retinal vein occlusion

A

Tortuosity and dilatation of branch of central retinal vein with AV nipping

Multiple retinal haemorrhages

Microaneurysms + hard exudates

NB must exclude hyperviscosity!

How well did you know this?
1
Not at all
2
3
4
5
Perfectly
14
Q

Fundoscopy appearances: background diabetic retinopathy / maculopathy

A

Blot haemorrhages
Hard exudates
Microaneurysms
Circinate exudates

How well did you know this?
1
Not at all
2
3
4
5
Perfectly
15
Q

Fundoscopy appearances: pre-proliferative diabetic retinopathy

A

Retinal ischaemia = characteristic here

  • Multiple dot + blot haemorrhages
  • Cotton wool spots (ischaemic areas)
  • IRMA: intra-retinal microvascular abnormalities
  • New vessel formation on the disc
How well did you know this?
1
Not at all
2
3
4
5
Perfectly
16
Q

Fundoscopy appearances: proliferative diabetic retinopathy

A

New vessels on the disc (first sign) and elsewhere

Haemorrhages

Hard exudates

NB can get ‘myopic crescent’ at the edge of the disc with peripheral retinal pigment layer prominence (don’t confuse with retinitis pigmentosa!)

How well did you know this?
1
Not at all
2
3
4
5
Perfectly
17
Q

Fundoscopy appearances: pan-retinal laser photocoagulation

A

Multiple laser scars with areas of hyperpigmentation

Patient will likely have reduced peripheral vision and a degree of night blindness

How well did you know this?
1
Not at all
2
3
4
5
Perfectly
18
Q

Fundoscopy appearances: multiple retinal haemorrhages

A

Seen in both deep and superficial layers of the retina

Hyperviscosity states (polycythaemia, waldenstrom’s macroglobulinaemia, myeloma) can lead to this

How well did you know this?
1
Not at all
2
3
4
5
Perfectly
19
Q

Fundoscopy appearances: retinal detachment

A

Area of bullous retina showing area of elevation with fluid

Trauma or choroidal metastasis

Painless ‘curtain coming down’ over vision

How well did you know this?
1
Not at all
2
3
4
5
Perfectly
20
Q

Discussion: what is retinitis pigmentosa?

A

Inherited form of retinal degeneration characterised by loss of photoreceptors

Inherited via autosomal recessive or x-linked pattern

Loss of night vision and peripheral vision

Progressive condition: by the time patients are middle-aged they reach criteria to be registered as blind

Driving implications: pt needs to inform DVLA about diagnosis, DVLA have rules about best corrective acuity + visual field loss

How well did you know this?
1
Not at all
2
3
4
5
Perfectly
21
Q

Syndromes that include retinitis pigmentosa and other features to look for to identify them?

A

RP and..

Ataxic: Freidreich’s ataxia, abetalipoproteinaemia, Refsum’s disease, Kearns-Sayre syndrome

Deafness: Refsum’s disease, Kearns-Sayre, Usher’s disease

Ophthalmoplegia + ptosis + pacemaker: Kearns-Sayre

Polydactyly: Laurence-Moon-Biedl

Ichthyosis: Refsum’s disease

How well did you know this?
1
Not at all
2
3
4
5
Perfectly
22
Q

Ddx of fundoscopic appearances of retinitis pigmentosa?

A

Diabetic retinopathy

Laser treatment scars

Infections: toxoplasmosis, rubella

How well did you know this?
1
Not at all
2
3
4
5
Perfectly
23
Q

Conditions associated with retinitis pigmentosa?

A

Laurence-Moon-Biedl syndrome
Usher syndrome
Alstrom syndrome
Refsum disease
Kearns-Sayre syndrome

How well did you know this?
1
Not at all
2
3
4
5
Perfectly
24
Q

Ddx of bilateral visual loss?

A

Glaucoma
Cataracts
Diabetic retinopathy
Macular degeneration
Papilloedema
Leber’s optic neuropathy (rarer)

How well did you know this?
1
Not at all
2
3
4
5
Perfectly
25
Q

Ddx of the inherited ataxias?

A

Spinocerebellar ataxia (trinucleotide repeat disorder)

Friedreich’s ataxia
Ataxia-telangiectasia
Ataxia with oculomotor apraxia type 1
Ataxia with oculomotor apraxia type 2 (all autosomal recessive)

How well did you know this?
1
Not at all
2
3
4
5
Perfectly
26
Q

How is the diagnosis of hereditary haemorrhagic telangiectasia made?

A

Using the Curacao criteria: if 3 criteria are present then the diagnosis is definite. If 2 present, diagnosis is possible.

  1. Epistaxis that is spontaneous + recurrent
  2. Multiple mucocutaneous telangiectasia
  3. Visceral lesions: GI / hepatic / lung / cerebral AVMs
  4. FH in 1st degree relative with genetic testing to confirm diagnosis
How well did you know this?
1
Not at all
2
3
4
5
Perfectly
27
Q

Features of hereditary haemorrhagic telangiectasia?

A

Patient presenting with recurrent nosebleeds and / or anaemia.

Telangiectasis on face, lips and buccal mucosa.

Vascular dysplasia which causes telangiectasia.

Autosomal dominant inheritance.

Vascular malformations: pulmonary shunts and IC aneurysms which cause SAH.

Increased risk of GI bleed, epistaxis and haemoptysis.

How well did you know this?
1
Not at all
2
3
4
5
Perfectly
28
Q

Ddx of thyrotoxicosis?

A

Graves’ disease (most common)

Thyroiditis of any cause (de Quervain’s, radiation, post-partum, drug-related)

Toxic multinodular goitre

Toxic adenoma

Ectopic secretion

Excess levothyroxine replacement

Hashimoto’s: period of hyperthyroidism before hypo

How well did you know this?
1
Not at all
2
3
4
5
Perfectly
29
Q

Bloods to test in suspected hyperthyroidism?

A

Thyroid function tests:
T3/4

Autoantibodies:
Thyroid peroxidase (TPO)
Thyroglobulin
TSH-receptor antibodies

How well did you know this?
1
Not at all
2
3
4
5
Perfectly
30
Q

Simple patient explanation for hyperthyroidism

A

Overactive thyroid gland - too much thyroid hormone which causes bodily functions to speed up.

If thyroid is overactive, treat with a medication (carbimazole) to reduce level of thyroid hormone.

Should start to work within 2 weeks, most people take it for 1-2 years.

50% chance of cure, 50% chance of relapse.

Warn about smoking cessation and agranulocytosis.

Other options:
- Radioactive iodine: kills thyroid cells, 90% of people respond. Can worsen eye disease, can’t have kids for 6mo, can’t hold children / pets after
- Surgery: 90% chance of cure. Risks of nerve damage, low calcium, need thyroid replacement for life

How well did you know this?
1
Not at all
2
3
4
5
Perfectly
31
Q

Graves’ disease management?

A

Medical:
- Carbimazole / PTU
- Propranolol for symptoms
- Radioactive iodine

Surgical:
- Thyroidectomy

How well did you know this?
1
Not at all
2
3
4
5
Perfectly
32
Q

Investigations in suspected Graves’?

A

Bedside: ECG (AF)

Bloods: TFTs, thyroid autoantibodies, FBC, UE, CRP, pregnancy test

Imaging: USS thyroid, radioactive iodine uptake scan, echo (high output CCF)

How well did you know this?
1
Not at all
2
3
4
5
Perfectly
33
Q

Causes of hypothyroidism?

A

Autoimmune: Hashimoto’s + atrophic hypothyroidism

Iatrogenic: post-thyroidectomy, iodine, amiodarone, lithium, anti-thyroid drugs

Iodine deficiency: dietary

Genetic: Pendred’s syndrome (+ deafness)

How well did you know this?
1
Not at all
2
3
4
5
Perfectly
34
Q

Investigations in hypothyroidism?

A

Bedside: ECG (rule out pericardial effusion and ischaemia)

Bloods: TSH, T4, autoantibodies, short SynACTHen test to exclude Addison’s

Imaging: CXR (pericardial effusion and CCF)

How well did you know this?
1
Not at all
2
3
4
5
Perfectly
35
Q

What are the complications of hypothyroidism?

A

Cardiac: pericardial effusion, CCF

Neuro: carpal tunnel syndrome, proximal myopathy, ataxia

How well did you know this?
1
Not at all
2
3
4
5
Perfectly
36
Q

Management of hypothyroidism?

A

Thyroxine titrated to TSH suppression + clinical response.

NB: can precipitate angina and unmask Addison’s disease leading to adrenal crisis.

How well did you know this?
1
Not at all
2
3
4
5
Perfectly
37
Q

Pertinent features of a tiredness history

A

Tiredness:
- what do you mean by tiredness? physical / mental / both?
- timing: sudden / gradual / intermittent / progressing / stable?

Thyroid questions:
- weight gain, poor memory, low mood, hoarse voice, dry skin, hair loss, feeling cold, constipation, angina, muscle weakness, oedema, SOB, neck swelling

Other causes of tiredness qu’s:
- malignancy: weight loss / night sweats, bleeding
- heart failure
- COPD
- renal (CKD)
- neuro (myasthenia)
- endocrine (Addison’s, Cushing’s, diabetes)
- infection (TB, HIV)
- mental health, anxiety, depression

PMH: thyroid disorders + treatment, other AI conditions

FH: thyroid + AI conditions

DH: amiodarone, lithium, carbimazole, interferon alpha

SH: impact on life, smoking / alcohol / illicit drug use

How well did you know this?
1
Not at all
2
3
4
5
Perfectly
38
Q

Summarised presentation for a patient you suspect has RA?

A

This patient has a peripheral symmetrical deforming polyarthropathy.

The presence / absence of nodules suggests seropositive / negative RA.

Hand function was preserved / restricted by deformity and weakness.

There were / was no evidence of extra-articular manifestations: no CT syndrome, no ocular, cardiovascular, pulmonary, GI or neurological involvement

How well did you know this?
1
Not at all
2
3
4
5
Perfectly
39
Q

What are some of the extra-articular features of RA?

A

Eyes: scleritis / episcleritis / scleromalacia / keratoconjunctivitis sicca

Lungs: lung nodules, pulmonary fibrosis, MTX-related pulmonary fibrosis

Heart: constrictive pericarditis, pericardial effusions

Kidneys: nephrotic syndrome, membraneous GN, renal amyloidosis

Neuro: peripheral neuropathies, carpal tunnel syndrome

Spleen: splenomegaly + RA + neutropaenia (Felty’s)

How well did you know this?
1
Not at all
2
3
4
5
Perfectly
40
Q

Radiological features of rheumatoid arthritis?

A

‘SALP’

Soft tissue swelling
Articular erosions
Loss of joint space
Periarticular osteopenia

How well did you know this?
1
Not at all
2
3
4
5
Perfectly
41
Q

What could be some causes of anaemia in RA?

A

Anaemia of chronic disease

IDA from chronic NSAID use

Bone marrow suppression - DMARD use

Autoimmune haemolytic anaemia

Splenomegaly + RA + neutropaenia (Felty’s)

How well did you know this?
1
Not at all
2
3
4
5
Perfectly
42
Q

What investigations would you like to do if suspecting RA?

A

Bedside: peak flow, obs, history

Bloods: FBC (multifactorial anaemia), CRP/ESR (biomarker of inflammation)

Autoantibodies: rheumatoid factor (positive in 70%), anti-CCP antibodies

Imaging: hand and feet XRs, CXR if suspecting fibrosis (+/- HRCT)

How well did you know this?
1
Not at all
2
3
4
5
Perfectly
43
Q

Management options for rheumatoid arthritis?

A

Mainstay of medical management rests on early use of DMARD.

Methotrexate - 1st line. If contraindicated: leflunomide, sulfasalazine, hydroxychloroquine.

If patients fail to respond to 2 DMARDS OR if they have poor prognostic markers, then biologics to be added.
- TNF alpha (adalimumab)
- anti-CD20 (rituximab)
- T cell blocker (abatacept)

Steroids are advised as short-term bridging therapy when switching to DMARD.

Surgical options: arthroplasty, arthrodesis + synovectomy.

How well did you know this?
1
Not at all
2
3
4
5
Perfectly
44
Q

Ddx for rheumatoid arthritis?

A

Any of the seronegative spondyloarthropathies:
- psoriatic arthritis
- ankylosing spondylitis
- reactive arthritis
- enteropathic arthritis

How well did you know this?
1
Not at all
2
3
4
5
Perfectly
45
Q

HLA association for RA?

A

RA has a strong association with HLA-DR4.

How well did you know this?
1
Not at all
2
3
4
5
Perfectly
46
Q

Diagnosis criteria for RA?

A

A patient has RA if they have 3 or more of the American College of Rheumatology Criteria:

  • Morning stiffness >1h
  • Swelling of 3 or more joints
  • Swelling of PIP, MCP, wrist joints
  • Symmetrical joint swelling
  • Rheumatoid nodules
  • Presence of IgM rheumatoid factor
  • Radiographic erosions / periarticular osteopenia
How well did you know this?
1
Not at all
2
3
4
5
Perfectly
47
Q

What objective tool can be used to measure disease activity in RA?

A

Objective tools such as DAS28 (disease activity score), based on an assessment of:
- no. of tender joints
- no of swollen joints
- global pain score
- ESR/CRP

DAS28 score >5.1 = active disease
score <2.6 = remission

How well did you know this?
1
Not at all
2
3
4
5
Perfectly
48
Q

Classical exam findings for rheumatoid arthritis?

A

Symmetrical deforming polyarthropathy, typically affecting MCPJs and PIPs

Presence of rheumatoid nodules at elbows

How well did you know this?
1
Not at all
2
3
4
5
Perfectly
49
Q

Hand deformities seen in RA?

A

Palmar subluxation + ulnar drift of MCP

Swan-neck deformity: rupture of lateral slip of extensor in finger

Boutonniere: rupture of central slip in finger

Z-shape deformity of thumbs

Swelling + subluxation of ulnar styloid

Carpal tunnel scar, wasting of intrinsic muscles of the hands

+ pain, swelling, stiffness of affected joints

How well did you know this?
1
Not at all
2
3
4
5
Perfectly
50
Q

DVLA considerations for syncope?

A

The ‘3 P’s’: provocation, prodrome, postural - if all 3 present then likely benign cause and can continue driving.

Solitary syncope with no clear cause: 6 month ban

Clear cause that has been treated: 4 week ban

Recurrent seizures: must be fit-free for 1 year

How well did you know this?
1
Not at all
2
3
4
5
Perfectly
51
Q

Differential diagnosis of syncope?

A

Cardiac: brady/tachy, obstructive cardiac lesion (AS, MS, HOCM or PE)

Neuro: epilepsy, vertebrobasilar insufficiency

Orthostatic: postural hypotension (+ medication history)

Vasovagal: stress, cough, micturition, defecation

How well did you know this?
1
Not at all
2
3
4
5
Perfectly
52
Q

Investigations for a patient with syncope?

A

Bedside: ECG, tilt-table test if orthostatic hypotension suspected, L/S BP

Bloods: FBC, UE, LFT, CRP/ESR, TFTs, morning cortisol

Imaging: echo (structural heart defects / valves), EEG, CT/MR brain

Special: Holter monitor, loop recorder, EP study, exercise tolerance test

How well did you know this?
1
Not at all
2
3
4
5
Perfectly
53
Q

Management of syncopal episodes?

A

Dependant on cause.

Cardiac: pacemaker, ICD, revascularisation / valve surgery

Vasovagal: education on avoidance, isotonic muscle contraction

Orthostatic hypotension: salt / water replacement, support stockings, medication review, fludrocortisone / midodrine, SSRIs

Neuro: anti-epileptics

How well did you know this?
1
Not at all
2
3
4
5
Perfectly
54
Q

Types of AF?

A

Paroxysmal: <7 days, self-terminating

Persistent: >7 days, requires chemical or electrical cardioversion

Permanent: >1 year / no further attempts to restore sinus rhythm

How well did you know this?
1
Not at all
2
3
4
5
Perfectly
55
Q

Investigations for AF?

A

Confirmation of AF (12-lead ECG or 24h Holter)

Echo: for structural heart disease, LVH, LA size (>4cm, recurrence is high)

TFTs

How well did you know this?
1
Not at all
2
3
4
5
Perfectly
56
Q

Management of AF?

A

Ideally, rate control: beta-blockers, digoxin, pacemaker, AV node ablation

If rhythm control is warranted, either chemical or electrical cardioversion:
- If severe / unstable heart failure, amiodarone
- If hypertension / LVH / CAD / HF, sotalol
- If no structural heart disease, flecainide

Pulmonary vein isolation: saved for refractory, symptomatic patients

Anticoagulation: with warfarin or NOAC

Need to predict embolic risk using the CHADSVASc score

NB: patients hgh risk for bleeding + embolic complications should be considered for left atrial appendage occlusion to isolate the commonest source of thrombus in AF

How well did you know this?
1
Not at all
2
3
4
5
Perfectly
57
Q

What is the CHADSVASc score used for, and what are the components of it?

A

Used to predict systemic embolus risk in AF.

Cardiac failure = 1
HTN = 1
Age >75 = 2
Diabetes = 1
Stroke / TIA / embolus = 2
Vascular disease = 1
Age 65 - 74 =1
Female = 1

0 - low stroke risk, no anticoagulation
1 - medium, pt preference
2 - high risk, oral anticoagulation recommended

How well did you know this?
1
Not at all
2
3
4
5
Perfectly
58
Q

How do you assess bleeding risk in a patient with AF that you are considering for anticoagulation?

A

HASBLED or ORBIT score.

HASBLED:
- HTN = 1
- abnormal kidney / liver function = 1 for each
- stroke = 1
- bleeding = 1
- labile INR = 1
- elderly = 1
- drugs (NSAIDs / alcohol) = 1 for each

3 or above - high risk, avoid oral anticoagulation

How well did you know this?
1
Not at all
2
3
4
5
Perfectly
59
Q

4 grades of hypertensive retinopathy?

A

Grade 1: silver wiring
Grade 2: silver wiring + AV nipping
Grade 3: silver wiring + AV nipping + CWS + flame haemorrhages
Grade 4: silver wiring + AVN + CWS + flame haemorrhages + papilloedema

How well did you know this?
1
Not at all
2
3
4
5
Perfectly
60
Q

Investigations you wish to perform in a patient presenting with hypertension?

A

Evidence of end-organ damage:
- fundoscopy
- LVH on ECG
- U+Es (renal impairment)
- CXR (heart failure)
- echo (heart failure)

Exclude underlying cause:
- pregnancy test
- urinalysis / ACR (blood and protein)
- U+Es
- renin / aldosterone levels, plasma metanephrines

How well did you know this?
1
Not at all
2
3
4
5
Perfectly
61
Q

Causes of hypertension?

A

Essential: 94% - associated with age / obesity / salt / alcohol

Renal: 4% - underlying CKD

Endocrine: 1% - Conn’s, Cushing’s, acromegaly, phaeochromocytoma

Aortic coarctation

Pre-eclampsia (pregnancy)

How well did you know this?
1
Not at all
2
3
4
5
Perfectly
62
Q

Findings of papilloedema on fundoscopy?

A

Blurring of disc margins / elevation of disc / venous engorgement

Causes: raised ICP, SOL, IIH, CVST, malignant hypertension, CRVO

(presents with normal visual acuity but tunnel vision, bilaterally)

How well did you know this?
1
Not at all
2
3
4
5
Perfectly
63
Q

What is accelerated phase / malignant hypertension?

A

A medical emergency.

Treatment:
- Grade III + IV retinopathy + hypertension: bed rest, long-acting CCB, BP monitoring, gradual drop in BP
- If have encephalopathy / stroke / MI / LV failure: IV vasodilators + invasive BP monitoring.

Over-rapid correction can lead to watershed stroke.

How well did you know this?
1
Not at all
2
3
4
5
Perfectly
64
Q

What autoimmune syndromes can Addison’s disease form a part of?

A

The autoimmune polyglandular syndromes (type 2)

Autoimmune thyroid disease

T1DM

How well did you know this?
1
Not at all
2
3
4
5
Perfectly
65
Q

How do we define different types of adrenal insufficiency?

A

Primary and secondary.

Primary:
- problem with adrenal glands causing decreased cortisol production
- 80% Addison’s disease
- CAH, HIV, TB, adrenal adenomas

Secondary:
- due to impairment of pituitary gland to produce ACTH
- most common reason is exogenous steroid use
- could also be due to pituitary adenoma, hypothalamic tumour

How well did you know this?
1
Not at all
2
3
4
5
Perfectly
66
Q

Pathophysiology of Addison’s disease?

A

Lack of feedback inhibition by cortisol, leading to increased ACTH and increase melanocyte-stimulating hormone (and therefore increased pigmentation).

In 80% of cases, due to an autoimmune process.

Other causes = adrenal mass, adrenal TB, amyloidosis, adrenalectomy and Waterhouse-Freidrichsen syndrome (meningococcal sepsis + adrenal infarction).

How well did you know this?
1
Not at all
2
3
4
5
Perfectly
67
Q

Investigations to do in a case of suspected Addison’s disease?

A

8am cortisol: no morning elevation suggests Addison’s (unreliable)

Short SynACTHen test: excludes Addison’s if cortisol rises to adequate levels

Long SynACTHen test: diagnose Addison’s if cortisol does not rise to adequate levels

Adrenal imaging (in primary AD) and / or pituitary imaging (in secondary AD) with MR/CT

Bloods: decreased sodium, raised potassium, increased urea (dehydration), hypoglycaemia, adrenal autoantibodies, TFTs and eosinophilia

CXR: malignancy or TB

How well did you know this?
1
Not at all
2
3
4
5
Perfectly
68
Q

Management of Addison’s disease?

A

Acute (adrenal crisis)
- 0.9% saline rehydration IV + glucose
- Hydrocortisone IV 100mg 6 hourly
- Treatment may unmask diabetes insipidus
- Anti-TB treatment increases clearance of steroid, so higher doses needed

Chronic:
- Patient education: increase steroid dose if unwell, steroid card, medic alert bracelet
- Titrate maintenance hydrocortisone + fludrocortisone dose to levels / response

How well did you know this?
1
Not at all
2
3
4
5
Perfectly
69
Q

Management of pyrexia of unknown origin?

A

Fernandez et al, 2018

Supportive until cause is found - avoid early abx until identification of cause and avoid steroid trials.

Consider stopping all drugs + reinstituting them one by one.

Ensure patient is aware of the strong possibility of no diagnosis being made.

Do not perform serological testing if there is no history of exposure to the pathogen you are testing for.

How well did you know this?
1
Not at all
2
3
4
5
Perfectly
70
Q

Investigations for pyrexia of unknown origin?

A

Bedside: obs / other exams

Bloods: culture (+ extended cultures for HACEK organisms), thick and thin films (parasites), HIV test, CRP / ESR, autoantibodies, immunoglobulins, complement levels, CK (malignant hyperthermia)

Other: bone marrow aspirate, CSF, TOE, (looking for vegetations, aortic root abscess, myxoma as cause), PET-CT

How well did you know this?
1
Not at all
2
3
4
5
Perfectly
71
Q

Chest pain differential diagnosis?

A

Stable angina vs ACS

Pericarditis

Costochondritis

Aortic dissection

Respiratory cause / pleurisy

Gastro cause

Miscellanous: HSV, anaemia, sickle cell, thyroiditis, sarcoid, substance abuse, anxiety

How well did you know this?
1
Not at all
2
3
4
5
Perfectly
72
Q

Acute management of ACS?

A

DAPT (aspirin / clopidogrel), glycoprotein IIb / IIIa inhibitor (tirofiban) if high risk (TIMI score >4) + fondaparinux

Anti-anginal: GTN + beta-blocker

Risk modification: statin + ACEi

If positive trop - proceed to coronary angiography (angioplasty + stent vs CABG)

If negative trop - functional tests to confirm ischaemia:
- exercise stress test
- MIBI scan
- stress echo
- cardiac MRI

How well did you know this?
1
Not at all
2
3
4
5
Perfectly
73
Q

What is the TIMI risk score for unstable angina / NSTEMI?

A

Estimates mortality for patients with unstable angina or NSTEMI.

Age >65: 1 point
>3 risk factors: 1
Known CAD: 1
Taking aspirin on admission: 1
Severe angina: 1
Trop rise: 1
ST depression: 1

> 3 points = high mortality risk

How well did you know this?
1
Not at all
2
3
4
5
Perfectly
74
Q

Causes of neuropathic ulcers?

A

Diabetes mellitus
Tabes dorsalis
Syringomyelia

How well did you know this?
1
Not at all
2
3
4
5
Perfectly
75
Q

Investigations for leg ulcers of unknown aetiology?

A

Doppler USS

ABPI (0.8 - 1.2 is normal, <0.8 implies arterial insufficiency)

Arteriography

How well did you know this?
1
Not at all
2
3
4
5
Perfectly
76
Q

Management of leg ulcers?

A

Specialist nurse / TVN: wound care

Venous:
- 4 layer compression bandaging (if no PVD)
- varicose vein surgery

Arterial:
- angioplasty or vascular reconstruction
- amputation

How well did you know this?
1
Not at all
2
3
4
5
Perfectly
77
Q

What are the different types of neurofibromatosis?

A

Type 1: neurofibromas

Type 2: bilateral acoustic neuromas

Schwannomatosis: painful schwannomas develop on spinal and peripheral nerves

How well did you know this?
1
Not at all
2
3
4
5
Perfectly
78
Q

Causes of enlarged nerves and peripheral neuropathy?

A

Neurofibromatosis
Leprosy
Amyloidosis
Acromegaly
Refsum’s disease

How well did you know this?
1
Not at all
2
3
4
5
Perfectly
79
Q

Ddx of neurofibromatosis?

A

LEOPARD syndrome
Legius syndrome
Proteus syndrome
Macrodystrophia lipomatosa

How well did you know this?
1
Not at all
2
3
4
5
Perfectly
80
Q

Genetic cause of neurofibromatosis?

A

Neurofibromatosis type I is the most common of the three types - is caused by genetic changes in the NF1 gene located on chromosome 17 (17q11.2). This gene encodes for neurofibromin, which acts as a tumour suppressor gene.

NF1 - neurofibromas

NF2 - schwannomas are more common

NF2’s most characteristic symptom is hearing loss, due to pressures of tumours on the acoustic nerve. May also have headaches / dizziness / nausea.

How well did you know this?
1
Not at all
2
3
4
5
Perfectly
81
Q

Investigations you may want to perform in a patient with suspected neurofibromatosis?

A

Bedside: EEG

Bloods: genetic testing

Imaging: XR, MR/CT brain, slit-lamp exam

Other: histology from biopsy of neurofibromas

How well did you know this?
1
Not at all
2
3
4
5
Perfectly
82
Q

Clinical signs expected in neurofibromatosis?

A

2 or more cutaneous neurofibromas

Cafe au lait patches: 6 or more, >15mm diameter

Axillary freckling

Lisch nodules: melanocytic hamartomas of the iris

Visual acuity - optic glioma / compression

BP: hypertension (associated with RAS and phaeochromocytomas)

Resp: fine crackles (honeycombing of lung - ILD)

Neuropathy with enlarged, palpable nerves

How well did you know this?
1
Not at all
2
3
4
5
Perfectly
83
Q

What conditions can be associated with NF1?

A

Phaeochromocytoma (2%)
Renal artery stenosis (2%)

How well did you know this?
1
Not at all
2
3
4
5
Perfectly
84
Q

Complications associated with NF1?

A

Epilepsy
Sarcomatous change
Scoliosis
Cognitive delay

How well did you know this?
1
Not at all
2
3
4
5
Perfectly
85
Q

Pertinent points in sarcoidosis history taking

A

Eye problems: painful red eye / visual disturbance

Skin: bruised / tender shins (erythema nodosum)

Joint pain

Resp: SOB, cough, haemoptysis, wheeze, Lofgren’s syndrome (triad of fever, erythema nodosum and hilar lymphadenopathy)

CVS: chest pain, palps, syncope

Abdo: pain, renal stones, polyuria, thirst

Neuro: headache, face droop, swollen face glands (parotids - Heerfordt-Waldenström syndrome), weakness / numbness

Systemic: weight loss / night sweats

TB history: contacts, travel, place of birth, vaccinations, occupation

HIV risk assessment

PMH / FH / DH

Occupational: birds / farmers / metal / stone / pottery / pets

How well did you know this?
1
Not at all
2
3
4
5
Perfectly
86
Q
A
How well did you know this?
1
Not at all
2
3
4
5
Perfectly
87
Q

Questions to ask if a rash sounds or looks like erythema nodosum

A

Strep - sore throat?

Meds - COCP, abx, sulphonamides, phenytoin

IBD questions

Behcet’s: mouth / genital ulcers

Pregnancy

Travel history

How well did you know this?
1
Not at all
2
3
4
5
Perfectly
87
Q

Examination for suspected sarcoidosis?

A

Hands: joint swelling, pulse

Face: rash (lupus pernio), eyes (+ fundoscopy), parotids, facial nerve assessment, look in mouth at throat / tonsils

Neck: lymph nodes (including parotids)

Chest: listen to heart sounds, chest, look for rash and check for axillary LNs

Abdo: feel for hepatosplenomegaly and inguinal LNs

Legs: look for erythema nodosum

Neuro: power, peripheral neuropathy

How well did you know this?
1
Not at all
2
3
4
5
Perfectly
88
Q

Pathophysiology of sarcoidosis?

A

A multisystem, granulomatous disorder

Tends to affect northern europeans, females, 20-40y, black more often than white individuals

CXR is abnormal in 90% of patients

Serum ACE is used to monitor disease activity and response to treatment, but not for diagnosis

On bloods often see hypercalcaemia, high IgG, anaemia and thrombocytopaenia

How well did you know this?
1
Not at all
2
3
4
5
Perfectly
89
Q

Investigations for suspected sarcoidosis?

A

Bedside: obs, urine dip, 24h urine for calcium (if blood calcium high), slit lamp exam, visual acuity, fundoscopy, o2 sats + ambulatory oximetry, spirometry, ECG

Bloods: CRP / ESR, FBC (thrombocytopaenia), LFTs, ACE, calcium (raised as non-caseating granulomas secrete vit D), immunoglobulins, UEs, vitamin D, TFTs

Imaging: CXR (hilar lymphadenopathy), BAL / bronchoscopy (+ biopsy), HR-CT, echo, 24h Holter, USS abdomen, CT/MRI brain

How well did you know this?
1
Not at all
2
3
4
5
Perfectly
90
Q

What would you expect on investigations for sarcoidosis if positive?

A

HRCT: ground glass changes + beading

Spirometry: restrictive pattern

BAL: increased lymphocytes

Biopsy from bronchoscopy: non-caseating granulomas + wide alveolar septae

How well did you know this?
1
Not at all
2
3
4
5
Perfectly
91
Q

Name some causes of bihilar lymphadenopathy.

A

Sarcoidosis

Infection: TB, mycoplasma, HIV

Malignancy: lymphoma, lung cancer, metastasis

Pneumoconiosis: silicosis, berylliosis

How well did you know this?
1
Not at all
2
3
4
5
Perfectly
92
Q

Causes of erythema nodosum?

A

‘NODOSUM’

No - no cause / idiopathic

D - drugs: COCP, abx, sulphonamides, phenytoin

O - OCP

S - sarcoidosis / streptococcus

U - ulcerative colitis / Crohn’s disease

M - mycobacterium / maternity (pregnancy)

How well did you know this?
1
Not at all
2
3
4
5
Perfectly
93
Q

What is Lofgren’s syndrome?

A

A type of acute sarcoidosis.

Inflammatory disorder characterised by a triad of:

  • Hilar lymphadenopathy
  • Erythema nodosum
  • Arthritis
    (+ fever)

Is often HLA-DRB1*03 associated

How well did you know this?
1
Not at all
2
3
4
5
Perfectly
94
Q

What tests would be important if you suspect a diagnosis of erythema nodosum?

A

ASO titre (antistreptolysin titre - strep A)

TB tests (early morning urine / sputum for AFB)

Mycoplasma serology

Pregnancy test

Blood film / LDH

How well did you know this?
1
Not at all
2
3
4
5
Perfectly
95
Q

How is sarcoidosis managed?

A

NSAIDs for arthralgia and skin

Refer to respiratory and ophthalmology

Steroids are indicated if:
- bilateral hilar lymphadenopathy
- infiltrates
- lung fibrosis
- eye problems
- hypercalcaemia
- neurological / cardiac involvement

(Give 40mg prednisolone for 4-6 weeks, then weaning doses)

Monitor with serum ESR and ACE

Steroid-sparing agents: MTX, hydroxychloroquine, ciclosporin, cyclophosphamide, infliximab

Surgery:
- lung transplant
- pacemaker

How well did you know this?
1
Not at all
2
3
4
5
Perfectly
96
Q

Features of Ehlers-Danlos on examination?

A

Skin + joints: hyperextensible skin, fragile skin, bruises, scarring, scars on knees

Joint hypermobility + dislocation

Cardiac: mitral valve prolapse

Abdominal: scars
- aneurysm rupture / dissection
- bowel perforation / bleeding

How well did you know this?
1
Not at all
2
3
4
5
Perfectly
97
Q

Pathophysiology of Ehlers-Danlos syndrome?

A

Autosomal dominant inheritance

Defect in collagen, causing increased skin elasticity

No premature coronary artery disease (unlike PXE)

6 types (joint hypermobility most common)

Largely a clinical diagnosis

Pregnancy can be dangerous - PROM + PPH

Genetic testing and counselling

How well did you know this?
1
Not at all
2
3
4
5
Perfectly
98
Q

Management for Ehlers-Danlos syndrome?

A

MDT approach

PT to strengthen muscles

Splints / orthoses to prevent joint dislocations

Analgesia

Activity modification (no contact sports)

How well did you know this?
1
Not at all
2
3
4
5
Perfectly
99
Q

Differential diagnosis for stretchy skin and hypermobile joints?

A

Joint hypermobility syndrome

Ehlers-Danlos Syndrome

Osteogenesis imperfecta

Marfan’s syndrome

Pseudoxanthoma elasticum (PXE)

Nail-patella syndrome (NPS)

How well did you know this?
1
Not at all
2
3
4
5
Perfectly
100
Q

Pertinent features in the history for a patient with PXE?

A

Skin problems: ‘looks like smoker’s skin’, hereditary and chronic

Other issues:
- hyperextensible joints
- reduced visual acuity
- hypertension
- MI or CVA (strong hx of coronary artery disease)
- gastric bleed

Family history of PXE symptoms (skin issues and CAD)

How well did you know this?
1
Not at all
2
3
4
5
Perfectly
101
Q

Important elements of the exam in a patient with suspected pseudoxanthoma elasticum?

A

Skin: ‘plucked chicken skin’ appearance, loose folds at neck and axillae, with yellow pseudoxanthomatous plaques, skin laxity

Eyes: blue sclerae and macular degeneration

Cardiovascular: hypertension and mitral valve prolapse

How well did you know this?
1
Not at all
2
3
4
5
Perfectly
102
Q

Pathophysiology of PXE?

A

A genetic disease that causes mineralisation of elastic fibres in some tissues.

Affects skin, eyes and blood vessels.

Inheritance: 80% autosomal recessive (ABCC6 gene, chromosome 16)

Premature coronary artery disease

How well did you know this?
1
Not at all
2
3
4
5
Perfectly
103
Q

History features in suspected Henoch-Schonlein Purpura?

A

Triad: ‘PAA’

Purpuric rash (buttocks and legs)
Arthralgia
Abdominal pain

Precipitants: infections (strep, HSV, parvovirus B19), drugs (abx)

Complications: renal involvement (IgA nephropathy), hypertension

How well did you know this?
1
Not at all
2
3
4
5
Perfectly
104
Q

Pathophysiology of HSP?

A

Small-vessel vasculitis: IgA + C3 deposition

Normal or raised platelet count (distinguishes from other causes of purpura)

Children > adults, Male > female

How well did you know this?
1
Not at all
2
3
4
5
Perfectly
105
Q

Other skin manifestations of sarcoidosis?

A

Other skin manifestations:
- Nodules + papules: face / ears / nose / neck / Koebner’s phenomenon
- Lupus pernio: diffuse bluish / brown plaque with central small papules commonly affecting the nose

How well did you know this?
1
Not at all
2
3
4
5
Perfectly
106
Q

Causes of secondary hyperlipidaemia?

A

Hypothyroidism

Nephrotic syndrome

Alcohol

Cholestasis

How well did you know this?
1
Not at all
2
3
4
5
Perfectly
107
Q

Treatment of necrobiosis lipoidica diabeticorum?

A

Topical steroid and support bandaging.

Tight diabetic control does not help.

How well did you know this?
1
Not at all
2
3
4
5
Perfectly
108
Q

Skin features of diabetes?

A

Shins:
- necrobiosis lipoidica diabeticorum: well-demarcated plaques with waxy-yellow centre and red / brown edges.
- diabetic dermopathy: red/brown, atrophic lesions

Feet + legs:
- ulcers: arterial / neuropathic
- eruptive xanthomas: yellow papules on buttocks / knees
- granuloma annulare: flesh-coloured papules in annular configurations on dorsum of feet + fingers

Injection sites:
- lipoatrophy
- fat hypertrophy

Cutaneous infections

Signs of other AI diseases:
- vitiligo
- Addison’s disease
- PVD

How well did you know this?
1
Not at all
2
3
4
5
Perfectly
109
Q

Features of malignant melanoma?

A

‘ABCDE’ - appearance of lesion

Asymmetrical
Border irregularity
Colour - black, irregular pigmentation
Diameter >6mm
Enlarging

Metastases (draining LNs, hepatomegaly, bone tenderness)

How well did you know this?
1
Not at all
2
3
4
5
Perfectly
110
Q

Management of malignant melanoma?

A
  • Excision
  • Staged on Breslow thickness (maximal depth of tumour invasion into dermis)
  • Glass eye + ascites - think ocular melanoma!
How well did you know this?
1
Not at all
2
3
4
5
Perfectly
111
Q

Features of SCC?

A

Sun exposed areas (+ lips / mouth)

Actinic keratoses: pre-malignant red scaly patches

Varied appearance: keratotic nodule, polypoid mass, cutaneous ulcer

Other lesions / previous scars

Metastases

SCC in situ = Bowen’s disease

Biopsy suspicious lesions

How well did you know this?
1
Not at all
2
3
4
5
Perfectly
112
Q

Features of BCC?

A

Usually on face / trunk (sun-exposed areas)

Pearly nodule with rolled edge

Superficial telangiectasia

Ulceration in advanced lesions

Other lesions

How well did you know this?
1
Not at all
2
3
4
5
Perfectly
113
Q

Treatment of BCC?

A

Natural history: grows slowly over months, rarely metastasizes

Treatment:
- surgical excision +/- radiotherapy
- curettage / cryotherapy if superficial

How well did you know this?
1
Not at all
2
3
4
5
Perfectly
114
Q

Pathophysiology of Paget’s disease?

A

A condition involving cellular remodelling and deformity of >1 bones.

The structural changes cause the bones to weaken, causing deformity, pain, fracture or arthritis of joints.

A frequent component of multisystem proteinopathy.

SQSTMI + RANK genes are associated with Paget’s.

How well did you know this?
1
Not at all
2
3
4
5
Perfectly
115
Q

Causes of sabre tibia?

A

Paget’s
Osteomalacia
Syphilis

How well did you know this?
1
Not at all
2
3
4
5
Perfectly
116
Q

Complications of Paget’s disease?

A

Osteogenic sarcoma

Basilar invagination (cord compression)

Kidney stones

How well did you know this?
1
Not at all
2
3
4
5
Perfectly
117
Q

Causes of angioid streaks (in the retina)?

A

Paget’s

PXE

Ehlers-Danlos Syndrome

How well did you know this?
1
Not at all
2
3
4
5
Perfectly
118
Q

Investigations in suspected Paget’s disease?

A

Grossly elevated alk phosphatase, normal calcium and phosphate.

Radiology:
- ‘moth eaten’ on plain films (osteoporosis circumscripta)
- increased uptake on bone scan

How well did you know this?
1
Not at all
2
3
4
5
Perfectly
119
Q

Management of Paget’s disease?

A

Bisphosphonates
Calcitonin
Surgery (for fractures, degenerative arthritis, bone deformity)

How well did you know this?
1
Not at all
2
3
4
5
Perfectly
120
Q

Diabetic retinopathy stages?

A

Background: ‘HBM’
- hard exudates
- blot haemorrhages
- microaneurysms

Pre-proliferative: backround features PLUS
- cotton wool spots
- flame haemorrhages

Prolierative: pre-proliferative features PLUS
- neovascularisation
- panretinal photocoagulation scars (treatment)

How well did you know this?
1
Not at all
2
3
4
5
Perfectly
121
Q

Screening processes for diabetic retinopathy?

A

Annual screening for all patients with diabetes

Refer to ophthalmology if pre-proliferative retinopathy or changes near macula

Background retinopathy usually occurs 10-20y after diagnosis of diabetes

Young people w/ T1DM usually get proliferative retinopathy

Older patients with T2DM usually get exudative maculopathy

How well did you know this?
1
Not at all
2
3
4
5
Perfectly
122
Q

What are some indications for photocoagulation treatment in diabetic retinopathy?

A

Maculopathy

Proliferative and pre-proliferative diabetic retinopathy

How well did you know this?
1
Not at all
2
3
4
5
Perfectly
123
Q

Complications of proliferative diabetic retinopathy?

A

Vitreous haemorrhage
Traction retinal detachment
Neovascular glaucoma

How well did you know this?
1
Not at all
2
3
4
5
Perfectly
124
Q

Clinical signs of cataracts on fundoscopy?

A

Loss of red reflex

Cloudy lens

May have RAPD with normal fundi

Associations: myotonic dystrophy (with bilateral ptosis)

How well did you know this?
1
Not at all
2
3
4
5
Perfectly
125
Q

Causes of cataracts?

A

Congenital: Turner’s, rubella

Acquired: age (usually bilateral), diabetes, steroids, radiation, trauma, storage disorders

How well did you know this?
1
Not at all
2
3
4
5
Perfectly
126
Q

Treatment of cataracts?

A

Surgery (as an outpatient):
- Phacoemulsification with prosthetic lens implantation
- YAG laser capsulotomy

How well did you know this?
1
Not at all
2
3
4
5
Perfectly
127
Q

History-taking pertinent features for a patient presenting with worsening mobility / falls

A

Pre-morbid social history:
- independence / dependence (carer role + frequency)
- mobility issues - PD, stroke? walks unaided or with stick or frame
- falls recently? how many?
- where do they live (impt for discharge planning)

Precipitant:
- infection: urinary sx, pneumonia sx
- drug changes: benzos, diuretics, BP meds, antipsychotics, steroids, PD drug changes
- systemic enquiry - sites of pain

Legal:
- advanced directives / living will

How well did you know this?
1
Not at all
2
3
4
5
Perfectly
128
Q

Exam points for a patient presenting with worsening mobility or falls?

A

Ask to stand / walk unaided

Proximal LL strength: rise from chair

Gait: wide-based, ataxic, hemiplegic, shuffling

Romberg’s: sensory ataxia

Assess LLs: inspection, tone, power, reflexes, sensation, coordination

Postural hypotension: assess lying and standing BP (20mmHg drop in systolic BP after 2 mins is significant)

How well did you know this?
1
Not at all
2
3
4
5
Perfectly
129
Q

Causes of delirium?

A

‘PINCHME’

Pain
Infection
Nutrition
Constipation
Hydration
Medication
Environment

How well did you know this?
1
Not at all
2
3
4
5
Perfectly
130
Q

Name some delirium screening tools

A

4AT: alertness, AMT4, attention, acute change / fluctuating course

CAM: confusion assessment method - assess the presence, severity and fluctuation of 9 delirium features

How well did you know this?
1
Not at all
2
3
4
5
Perfectly
131
Q

Management of delirium?

A

Treat reversible causes: abx, avoid polypharmacy

MDT: nurse, social worker, OT/PT
- mobility aids
- home modifications / residential care
- resus decision / ceiling of care

How well did you know this?
1
Not at all
2
3
4
5
Perfectly
132
Q

What is the FRAT score used for?

A

Indicates a patient’s risk of falling and guides further management for worsening mobility / falls.

  1. Falls in the last year
  2. 4+ meds per day
  3. Dx stroke / PD
  4. Balance issues
  5. Can they get up from a chair without using arms

<3 points = lower risk
>3 points = higher risk

How well did you know this?
1
Not at all
2
3
4
5
Perfectly
133
Q

What is the FRAX tool used for?

A

FRAX predicts the 10-year probability of a patient having a major osteoporotic fracture.

How well did you know this?
1
Not at all
2
3
4
5
Perfectly
133
Q

Investigations for a patient with worsening mobility / falls?

A

Bedside: obs, collateral history, urine MCS / CSU, delirium screen 4AT or CAM

Bloods: septic screen, blood cultures, bone profile and UEs (elecs), FBC, CRP (infection)

Imaging: CXR (infection), if fall and confusion / reduced consciousness, consider CT head non contrast. High risk of subdural in atrophic brain

How well did you know this?
1
Not at all
2
3
4
5
Perfectly
134
Q

Which medications would we worry about as a cause of falls in polypharmacy?

A

Antihypertensives (low BP)

Sedatives

Diuretics (elec abnormalities and low BP)

Antidepressants (hyponatraemia)

How well did you know this?
1
Not at all
2
3
4
5
Perfectly
135
Q

Which medications add to anticholinergic burden?

A

‘PC SOAP’

Promethazine
Cetirizine
Solifenacin
Oxybutynin
Amitriptyline
Prochlorperazine

How well did you know this?
1
Not at all
2
3
4
5
Perfectly
136
Q
A
How well did you know this?
1
Not at all
2
3
4
5
Perfectly
137
Q

Management of TIA?

A

Once structural causes have been ruled out with brain imaging, mgmt focuses on secondary prevention of further episodes.

Aspirin 300mg OD for 2 weeks, then clopidogrel 75mg OD lifelong.

Optimise BP, diabetes, cholesterol, diet, exercise, smoking cessation, reduce alcohol.

Surgical: carotid endarterectomy within 2 weeks if symptomatic carotid stenosis (70-99%) (ESCT criteria).

Anticoagulate if AF present.

DVLA: no driving for 1 month.

How well did you know this?
1
Not at all
2
3
4
5
Perfectly
138
Q

Ddx of TIA?

A

Hemiplegic migraine
Hypoglycaemia
Seizure
Syncope
GCA
Electrolyte disturbances

How well did you know this?
1
Not at all
2
3
4
5
Perfectly
139
Q

List some causes of CVA in younger patients?

A

Antiphospholipid syndrome

Extracranial dissection

Vasculitis

Substance abuse: cocaine / methamphetamines

Metabolic / mitochondrial disease

Sickle cell disease

How well did you know this?
1
Not at all
2
3
4
5
Perfectly
140
Q

Features of antiphospholipid syndrome?

A

Consider APLS if young with TIA + recurrent miscarriages.

Requires anticoagulation: current evidence favours warfarin > DOAC.

Consider bubble echo looking for PFO in young patient.

Complications: TIA / stroke, DVT / PE, livedo reticularis, thrombocytopaenia, thrombophlebitis, recurrent miscarriages.

How well did you know this?
1
Not at all
2
3
4
5
Perfectly
141
Q

Pertinent examination points in suspected TIA

A

Visual acuity
Visual fields
Pupillary responses
Fundoscopy

H-test for eye movements
Palpate temporal arteries

Cardiovascular exam

Neuro: pronator drift, power, sensation, finger-nose testing, reflexes, speech, gait

How well did you know this?
1
Not at all
2
3
4
5
Perfectly
142
Q

Pertinent TIA history points

A

Timing: when, sudden / gradual, how long, complete recovery now? previous episodes

Vision: painful / painless, blurred, distorted, bits missing, double vision

Neuro: weakness, speech, swallow, headache

GCA: headache / scalp / jaw pain

Migraine: aura, headache, sensitive to light / noise, N+V

Seizure: LOC, drowsy after, incontinence, tongue biting

Hypoglycaemia: sweating, palps, hunger before

PMH stroke / TIA / recurrent miscarriages. CV risk factors: HTN, diabetes, high cholesterol, smoking, MI, stroke

DH / FH

SH: alcohol, drugs, smoking, CV risk factors

How well did you know this?
1
Not at all
2
3
4
5
Perfectly
143
Q

Examination features for suspected eczema?

A

Rash:
- erythematous + lichenified patches of skin
- mainly flexor aspects of joints
- fissures (hands, feet. painful)
- excoriations
- secondary bacterial infection

Associated atopy:
- respiratory exam: polyphonic wheeze of asthma

Investigations:
- patch testing for allergies

How well did you know this?
1
Not at all
2
3
4
5
Perfectly
144
Q

Management of eczema?

A

Avoid precipitants

Topical: emollients, steroids, tacrolimus

Antihistamines for itch

Abx for secondary infection

UV light therapy

PO prednisolone in severe cases

How well did you know this?
1
Not at all
2
3
4
5
Perfectly
145
Q

Causes of nail pitting?

A

Psoriasis
Lichen planus
Alopecia areata
Fungal infections

146
Q

Examination pertinent points in a patient with psoriasis

A

Skin:
- chronic plaque type: multiple well-demarcated, pink scaly plaques on extensor surfaces
- check behind ears, scalp + umbilicus
- Koebner phenomenon: plaques at sites of trauma
- skin staining from coal tar treatment

Nails:
- pitting, onycholysis, hyperkeratosis, discolouration

Joints:
- psoriatic arthropathy

147
Q

Definition of psoriasis?

A

Epidermal hyperproliferation and accumulation of inflammatory cells

148
Q

Management options for psoriasis

A

Topical:
- emollients
- calcipotriol
- coal tar (stains)
- dithranol
- hydrocortisone

Phototherapy:
- UVB
- psoralen + UVA (PUVA)

Systemic:
- cytotoxics (MTX / ciclosporin)
- anti-TNF (adalimumab)
- retinoids (acitretin)

149
Q

What is SLE?

A

A multisystem, inflammatory autoimmune disorder.

Broad spectrum of clinical presentations, affects all organs and tissues.

Demographics: Female > male, African + Latin americans, Afro-caribbeans, South Asians and Hispanic, 16-55y.

Takes a chronic, waxing and waning course.

150
Q

How is SLE diagnosed?

A

Initially the American College of Rheumatology criteria (out of 11), and now the SLICC (2012) classification criteria (out of 17)

  • 4/17 including 1 clinical and 1 immunological
  • biopsy-proven lupus nephritis and ANA / anti-dsDNA
151
Q

Management options for SLE?

A

Mild disease (cutaneous / joint only):
- topical corticosteroids
- hydroxychloroquine

Moderate disease (+ other organ involvement):
- prednisolone PO
- azathioprine

Severe disease (+ severe involvement of vital organs)
- methylprednisolone
- MMF (for lupus nephritis)
- cyclophosphamide
- azathioprine

152
Q

What antibodies are SLE-specific?

A

ANA
anti-dsDNA
elevated immunoglobulins

153
Q

Investigations to consider in a patient if SLE is suspected?

A

Bedside: obs, sats, ECG, skin/renal biopsy, nailfold capillaroscopy, pregnancy test

Bloods:
- FBC, UE, LFT, clotting, ESR / CRP
- ANA, RF, anti-CCP, ANCA, immunoglobulins, anti-dsDNA
- anti-Scl70, anticentromere, anti Ro/La, anti-Jo1, antihistone
- HIV / Hep B + C / CMV / EBV screen
- serum electrophoresis + urine BJ proteins
- ACE (sarcoid), ferritin (Still’s), anti-GBM (Goodpastures)

Other:
- If suspecting specific organ involvement, relevant tests for that organ

154
Q

Drugs that can cause drug-induced lupus?

A

Hydralazine
Procainamide
Isoniazid
Phenytoin
Interferon

155
Q

Which antibodies are specific to certain rheumatological conditions?

A

SLE: ANA, dsDNA, anti-Smith

Antiphospholipid syndrome: anti-cardiolipin, lupus anticoagulant, beta2-glycoprotein

Sjogren’s syndrome: anti-Ro (SSA), anti-La (SSB)

MCTD: anti-U1 RNP

RA: rheumatoid factor, CCP

Myositis: anti-Jo1

Scleroderma: anti-centromere, Scl-70, RNA polymerase III

ANCA vasculitis: c-ANCA (GPA), p-ANCA (EGPA)

156
Q

New GOLD (2023) COPD management guidelines?

A

Severity of COPD is categorised on MRC dyspnoea scale, instead of previously-used FEV1.

0-1 exacerbations (not leading to hospital admission):
- mMRC 0-1, CAT <10 = single bronchodilator
- mMRC =>2, CAT =>10 = LABA + LAMA

=>2 mdoerate exacerbations or =>1 leading to hospitalisaton:
= LABA + LAMA
(consider LABA + LAMA + ICS if blood eosinophil count >300)

Key:
mMRC: modified Medical Research Council dyspnoea questionnaire
CAT: COPD Assessent Test

157
Q

What questions to ask in an extra-articular features rheumatology screen? (ie if suspect SLE / MCTD / Sjogrens / scleroderma etc)

A

Systemic: weight loss, night sweats, fatigue, fever

Rashes: sensitive to sun? hair loss? mouth / nose ulcers? swollen / tight skin?

Eyes: Dry eyes / mouth, visual changes, painful eyes

CVS / resp: CP, SOB, orthopnoea, palps, ankle swelling, LOC, wheeze, cough, haemoptysis, exercise tolerance

Raynaud’s: fingers go cold + change colour (white, blue then red)

GI / GU: bladder and bowels, dysphagia, heartburn, abdo pain

Renal: high BP, blood / protein in urine

Neuro: headaches, double vision, LOC, dizziness, weakness, numbness

Muscle weakness / pain

Pregnancy: baby have heart block or rash?

PMH/DH: drug-induced lupus
FH of rheumatological or AI disease
SH: pregnant, breastfeeding? plans to do so?

158
Q

Important examination features in suspected SLE?

A

Head / face / scalp: malar rash, discoid rash, oral ulceration, scarring alopecia (edges of bald patches look more ragged than alopecia areata)

Hands / nails: nailfold vasculitis, synovitis, Raynaud’s, fistula on arm, Jaccoud’s arthropathy (mimics RA but due to tendon contractures and not joint destruction)

CVS exam

Resp exam: effusion, rub, fibrosing alveolitis

Abdo: organomegaly, renal transplant

Legs: oedema, DVT

Skin: discoid lupus, vasculitic rash, photosensitivity rash, livedo reticularis

Cushingoid appearance: steroid use

159
Q

Investigations in systemic sclerosis?

A

Autoantibodies (bloods):
- ANA +ve in 90%
- anti-centromere in limited SS
- anti-Scl70 in diffuse SS

Hand XRs: for calcinosis.

Pulmonary: CXR, HR-CT, PFTs. Expect lower lobe fibrosis and aspiration pneumonia.

GI: Contrast scans, FBC, B12 and folate. Gut dysmotility and malabsorption.

Renal: UE, urinalysis, urine microscopy (casts), consider renal biopsy.

Cardiac: ECG and echo. Myocardial fibrosis and arrhythmias.

160
Q

What is CREST syndrome?

A

CREST is a type of limited systemic sclerosis.
It consists of:
- Calcinosis
- Raynaud’s phenomenon
- Esophageal dysmotility
- Sclerodactyly
- Telangiectasia

161
Q

What is systemic sclerosis?

A

SS is a connective tissue disease characterised by autoimmunity, vasculopathy and fibrosis.

Most commonly affects females aged 35-65y.

Prognosis for diffuse SS is around 50% 5-year survival, most deaths are due to respiratory failure.

162
Q

Classification of systemic sclerosis?

A

Limited SS:
- distribution below elbows, below knees and face
- slow progression (years)

Diffuse SS:
- widespread cutaneous and early visceral involvement
- rapid progression (months)

163
Q

Management of systemic sclerosis?

A

Symptomatic treatments:
- camouflage creams
- Raynaud’s treatments: hand warmers, CCBs, ACEi’s, prostacyclin infusions

Renal: ACE’s

GI: PPI for reflux

Treat pulmonary hypertension: CCBs, diuretics, iloprost, bosentan, sildenafil, o2

Immunosuppression: MMF, MTX, cyclophosphamide, azathioprine, ciclosporin, prednisolone, rituximab

164
Q

Pertinent features of exam for someone with suspected systemic sclerosis?

A

Hands:
- sclerodactyly ‘prayer sign’
- calcinosis (may ulcerate)
- assess function: hold pen

Face:
- tight skin
- microstomia
- beaked nose
- peri-oral furrowing, telangiectasia, alopecia

Other skin lesions:
- morphoea: focal / generalised patches of sclerotic skin
- en coup de sabre (scar down central forehead)

BP: hypertension

Resp: fibrosis (fine, bibasal crackles)

Cardiac: pulmonary hypertension (RV heave, loud P2, TR), evidence of CCF, pericarditis (rub)

165
Q

Management of ankylosing spondylitis?

A

MDT approach!

  • PT and hydrotherapy
  • Analgesia: NSAIDs, paracetamol, weak opioids
  • Corticosteroid injections: for sacroiliitis and enthesitis
  • Biological agents: anti-TNF (adalimumab)
  • Treat complications such as iritis
  • Treat osteoporosis with bisphosphonates (prevent spinal fractures)
  • Smoking cessation (close association with disease activity)
166
Q

What pre-treatment investigations must be considered before starting anti-TNF?

A

Immunisations

Screen for active and latent TB with CXR and blood test

167
Q

How is ankylosing spondylitis diagnosed?

A

Mainly from clinical history and examination with supporting radiological evidence.

Young patients (<40) with +ve FH

Plain spine XR: erosions / sclerosis of SI joints, and squaring of vetebrae (‘bamboo spine’)

Bloods: raised ESR / CRP, anaemia of chronic disease

Genetic testing: HLA-B27

CXR / HRCT if suspecting lung fibrosis, consider PFTs

168
Q

What are the extra-articular manifestations of ankylosing spondylitis?

A

The 5 A’s:

Anterior uveitis (commonest, 30%)

Apical lung fibrosis

Aortic regurgitation (midline sternotomy)

AV node block (pacemaker)

Arthritis (may be psoriatic arthropathy)

169
Q

What are the immunological associations with ankylosing spondylitis?

A

Seronegative spondyloarthropathy

HLA-B27 positive in >90% of individuals

TNF and interleukin-1 also implicated in disease activity

170
Q

How do you differentiate on PFTs between mechanical restriction and interstitial lung disease?

A

Look at the TLCO (transfer factor).

Mechanical restriction - normal TLCO
ILD - reduced TLCO due to underlying damaged lung

171
Q

DDx of breathlessness in ankylosing spondylitis?

A

Apical lung fibrosis

Anaemia: NSAIDs / anaemia of chronic disease

Mechanical restriction due to fusion of the spine causing restrictive defect

Cardiac causes: AR

172
Q

How is ankylosing spondylitis disease activity measured?

A

Using the BASDAI: Bath Ankylosing Spondylitis Disease Activity Index.

  • 10-point scale
  • anything >4 indicates active disease
173
Q

Pertinent examination points in suspected ankylosing spondylitis?

A

Spine:
- ask pt to stand up (back and chest exposed)
- kyphotic spine, hyperextension of the neck (question-mark posture) - increased occiput to wall distance
- reduced spinal movements
- increased AP diameter of chest wall
- Schober’s test: 2 points marked 15cm apart, expand by <5cm on maximum forward flexion is a positive test

Cardiac:
- listen to aortic area and left sternal edge (AR)

Chest:
- apical fibrosis, apical fine creps

Eyes: iritis, visual acuity check

Gait: likely antalgic, will make spinal deformity more obvious

174
Q

What is the difference between Cushing’s disease and Cushing’s syndrome?

A

Cushing’s disease: glucocorticoid excess due to an ACTH-secreting pituitary adenoma

Cushing’s syndrome: the physical signs of glucocorticoid excess

175
Q

Investigations to perform in suspected case of Cushing’s syndrome?

A
  1. Confirm high cortisol
    - 24h urinary collection
    - low dose dex suppression test (won’t suppress cortisol if +ve)
  2. If elevated cortisol, identify cause
    - ACTH level:
    if high, ACTH-secreting tumour / pituitary adenoma.
    If low, adrenal adenoma / carcinoma.
  3. Other tests
    - MRI pituitary fossa +/- adrenal CT +/- whole body CT to identify lesion
    - Inferior petrosal sinus vein sampling (pituitary vs ectopic origin)
175
Q

Management of Cushing’s

A

Surgical:
- transsphenoidal approach to remove pituitary tumour
- adrenalectomy for adrenal tumours

Nelson’s syndrome: bilateral adrenalectomy causes Cushing’s disease, as causes increased ACTH production and leads to pituitary overgrowth due to lack of feedback inhibition

Pituitary irradiation

Medical: metyrapone

176
Q

Ddx of proximal myopathy causes?

A

Inherited: muscular / myotonic dystrophy

Endocrine: Cushing’s, hyperparathyroidism, thyrotoxicosis

Inflamm: polymyositis, RA

Metabolic: osteomalacia

Malignancy: paraneoplastic / LEMS

Drugs: alcohol, steroids

177
Q

Pertinent examination points for suspected Cushing’s syndrome?

A

Spot diagnosis:
- Face: round, hirsuite, acne
- Skin: bruised, thin, purple striae
- Back: intrascapular fat pad
- Abdomen: central adiposity
- Legs: wasting and oedema

Complications:
- HTN (check BP)
- Diabetes (CBG)
- Osteoporosis (kyphosis)
- Cellulitis
- Proximal myopathy (stand from sitting)

Cause:
- Exogenous: signs of chronic condition (RA, COPD) requiring steroids
- Endogenous: bitemporal hemianopia + hyperpigmentation (if increased ACTH)

178
Q

Why is OSA associated with acromegaly?

A

Acromegaly causes soft tissue swelling in the face / around the throat.

This can lead to respiratory compromise - especially when lying down at night.

179
Q

Management of acromegaly?

A

1st line: surgery - transsphenoidal resection (curative)

2nd line: somatostatin analogues (ocretotide), bromocriptine if not tolerated

3rd line: radiotherapy to pituitary gland is an option if other treatments fail

NB a potential complication of surgery is panhypopituitarism - need to warn patients about this before surgery.

180
Q

Why do acromegalic patients get visual disturbance?

A

Classically, a bitemporal hemianopia occurs.

This is due to compression by the adenoma on the optic chiasm.

181
Q

Clinical findings in a patient with acromegaly?

A

General appearance: prognathism, prominent supraorbital ridges, coarse facial features, macroglossia, incrased teeth separation, spade-like hands, sweating, large feet

May have goitre

Bitemporal hemianopia on VF testing

Carpal tunnel syndrome

Axillae for acanthosis nigricans

Chest: gynaecomastia, galactorrhoea

Cardiovascular: increased JVP, displaced apex beat, bibasal crackles, pedal oedema (CCF)

Joints: arthropathy (Heberden’s nodes)

Offer to examine: BP, urine dip (glucose), DRE (polyps), testicular exam (hypogonadism)

182
Q

What investigations would you request in a case of suspected acromegaly?

A

Screening: test IGF-1 levels (will be high)

Confirm diagnosis with OGTT (shows failure to suppress GH)

Look for underlying cause: MRI brain, looking for pituitary adenoma

Investigating complications:
- fasting glucose + HbA1c
- anterior pituitary hormone screen
- ECG: look for LVH and cardiac conduction defects
- TTE: assess ejection fraction
- sleep study: assess for OSA
- colonoscopy for polyps
- NCS for carpal tunnel syndrome

183
Q

Management of the complications of acromegaly?

A

OSA: CPAP

CCF: diuretics, ACEi’s, beta blockers

Diabetes: hypoglycaemic agents

Anterior pituitary deficiency: hormone replacement

184
Q

What is the underlying disease mechanism for osteogenesis imperfecta?

A

A problem with poorly formed type 1 collagen.

In >90% of cases, OI occurs due to mutations in COLIA1 or COLIA2 genes.

185
Q

Inheritance pattern of osteogenesis imperfecta?

A

Predominantly inherited in an autosomal dominant pattern.

There are 8 different variations of OI, can develop spontaneously - this is the more severe form of the condition.

186
Q

Clinical features of osteogenesis imperfecta?

A

Blue discolouration of eyes

Recurrent fractures since birth

Palpitations + abdominal discomfort

Short stature, barrel chest

Deformity of the legs

Abnormal gait / joint hypermobility / skin laxity

Discoloured, translucent teeth (dentogenesis imperfecta)

Hearing impairment (middle ear bones affected)

187
Q

Investigations for suspected osteogenesis imperfecta?

A

Bloods: bone profile, vitamin D, genetic screening (predominantly an inherited condition)

Imaging: x-rays and DEXA to look at bone density. Echocardiogram.

NB: echo is requested as there is an association with bicuspid aortic valve in patients with OI, and they can also develop AR.

188
Q

Management for osteogenesis imperfecta?

A

Depends on the results of relevant investigations for OI.

Replace low calcium / phosphate / vit D levels.

If DEXA shows decreased bone density, consider bisphosphonates.

Orthotics review.

PT review.

189
Q

Main clinical features of LEMS?

A

Diminished reflexes that become brisker after exercise.

LL girdle weakness.

Associated with malignancy (SCLC).

Antibodies to pre-synaptic calcium channels.

EMG shows ‘second wind’ phenomenon on repetitive stimulation.

190
Q

Causes of bilateral extra-ocular palsies

A

Myasthenia gravis
Graves’ disease
Mitochondrial cytopathies (CPEO)
Miller-Fisher variant of GBS
Cavernous sinus pathology

191
Q

Management of myasthenia gravis?

A

Acute: IV immunoglobulin or plasmapharesis

Chronic:
- ACh esterase inhibitor (pyridostigmine)
- Immunosuppression: steroids / azathioprine
- Thymectomy is beneficial, even if patient does not have a thymoma

192
Q

What causes myasthenia gravis?

A

Anti-nicotinic ACh receptor antibodies affect neurotransmission.

193
Q

Investigations for myasthenia gravis?

A

Diagnostic tests:
- Anti-ACh receptor antibodies (+ve in 90%)
- Anti-MuSK antibodies
- EMG: decremented response to impulses
- Tensilon / edrephonium test: not done anymore as causes heart block
- CT / MRI chest (thymoma in 10%)
- TFTs (Graves’ in 5%)

194
Q

Causes of bilateral facial nerve palsy?

A

Guillain-Barre Syndrome
Sarcoidosis
Lyme disease
Myasthenia gravis
Bilateral Bell’s palsy

195
Q

Bell’s palsy features and management?

A

Bell’s palsy is the commonest cause of facial nerve palsy.

Rapid onset over a few days, HSV1 has been implicated. Induced swelling and compression of the nerve within the facial canal.

Management:
- Prednisolone started within 72h (+ aciclovir if severe)
- Eye protection (artificial tears, tape shut at night)
- Prognosis: 70-80% make a full recovery
- Pregnancy: Bell’s palsy is more common, outcome may be worse

196
Q

Causes of 7th nerve palsy?

A

Bell’s palsy
Ramsay-Hunt syndrome
Mononeuropathy (diabetes, sarcoid)
Tumour / trauma
MS / stroke

197
Q

What is Bell’s phenomenon?

A

The eyeball rolls up on attempted eye closure (seen in Bell’s palsy)

198
Q

How to work out the level of the lesion in a facial nerve palsy?

A

Pons: + VI palsy and long tract signs (MS+stroke)

Cerebello-pontine angle: + V, VI, VIII + cerebellar signs (tumour)

Auditory / facial canal: +VIII (cholesteatoma or abscess)

Neck / face: + scars or parotid mass (tumour or trauma)

199
Q

Clinical signs of retinal artery occlusion?

A

Pale, milky fundus with thread-like arterioles

+/- cherry red spot on macula (choroidal blood supply)

Cause: AF or carotid stenosis with bruit

200
Q

Causes of central retinal artery occlusion?

A

Embolic: carotid plaque rupture / cardiac mural thrombus

GCA: tender scalp + pulseless temporal arteries

201
Q

Clinical features of age-related macular degeneration?

A

Signs:
- wet (neovascular, exudative) or dry (non-neovascular, atrophic, non-exudative)
- Macular changes: drusen, geographic atrophy, fibrosis, neovascularisation (if wet)

Symptoms:
- loss of CENTRAL vision
- issues reading / driving / recognising faces

Management:
- ophthalmology referral
- wet ARMD: intravitreal anti-VEGF injections

202
Q

Features of retinal vein occlusion?

A

Clinical signs:
- flame haemorrhages ++ radiating out from swollen disc
- engorged, tortuous veins
- cotton wool spots

Effect: rubeosis iridis causes secondary glaucoma in CRVO - visual loss or visual field defect

203
Q

Causes of central retinal vein occlusion?

A

HTN

Hyperglycaemia / diabetes

Hyperviscosity syndromes: Waldenstrom’s macroglobulinaemia or myeloma

Increased intraocular pressure (glaucoma)

204
Q

Causes of a pale optic disc?

A

‘PALE DISCS’

Pressure: tumour, glaucoma (most common cause)

Ataxia: Freidreich’s

LEber’s optic neuropathy

Dietary: B12, Degenerative: RP

Ischaemia: CRAO

Syphilis and other infections: CMV + toxoplasmosis

Cyanide + other toxins: alcohol, lead, tobacco

Sclerosis: MS (most common cause)

205
Q

Management of sickle cell crisis?

A

Oxygen +/- CPAP

IV fluids, analgesia

Abx if evidence of infection

Blood transfusion / exchange transfusion depending on severity of crisis

Hydroxycarbamide / exchange transfusion programme if frequent crises / poor prognosis

Long-term folic acid + penicillin (due to hyposplenism)

206
Q

Ddx for a patient with sickle cell presenting with fatigue / breathlessness / bone pain / pleuritic chest pain

A

Vaso-occlusive sickle crisis

Acute sickle chest crisis

Sicke cell crisis + pulmonary hypertension

Vaso-occlusive crisis with PE

207
Q

Types of sickle cell crisis?

A

Vaso-occlusive crisis (sickling in small vessels of any organ). Often precipitated by viral illness, exercise or hypoxia.

Leg ulcers due to ischaemia.

Chest crises (high mortality associated with these in adulthood).

Priapism.

Pulmonary hypertension.

208
Q

Differential diagnosis for diarrhoea?

A

Infectious:
- viral: norovirus, rotavirus
- bacterial: e.coli, salmonella, shigella, campylobacter, v.cholerae, c.difficile
- parasites: giardia, amoebic dysentry

IBD: UC or Crohn’s - blood, weight loss, abdominal pain, mucus

Colorectal Ca: blood, weight loss, enlarged LNs, abdominal masses

Coeliac disease: symptoms triggered by certain foods (bread, pasta)

Medication-related: abx, cytotoxics, laxatives, PPIs, digoxin, NSAIDs, beta-blockers

Thyrotoxicosis: sweating, hot, bulging eyes, neck lump, anxious

Bacterial overgrowth: post-gastroenteritis, burping / passing excess wind

Pancreatic insufficiency: steatorrhoea

Short bowel syndrome: previous bowel surgeries?

Ischaemic colitis: history of stroke / AF / cardiovascular disease?

209
Q

Appropriate investigation / management for a patient with unexplained diarrhoea?

A

Bedside: stool sample, fluid resus. Send stool sample for:
- MC+S
- C.diff toxin
- Ova, cysts and parasites
- Faecal calprotectin

Bloods: cultures, FBC, WCC, CRP, UE, VBG (lactate), LFTs, TFTs, autoimmune screen, TTG (coeliac). Hep A/B/C and HIV.

Imaging: AXR (toxic megacolon / perforation)

Consider:
- Antibiotics + IV steroids (if suspect an IBD flare)
- Flexi-sig / colonoscopy / OGD + biopsies
- Abdo USS
- CT AP
- Faecal elastase (pancreatitis)

210
Q

Features of a spot diagnosis for a patient with Down’s syndrome?

A

Low-set ears

Epicanthic folds

Protruding tongue

Collapsed nasal bridge

Single palmar crease

Wide gap between 1st and 2nd toes

Cardiac murmurs

Reduced IQ

211
Q

Features of a spot diagnosis for a patient with acromegaly?

A

Face: prominent supraorbital ridges, large lips and nose, prognathism, spaced teeth

Macroglossia, large sweaty hands

Complications: carpal tunnel, diabetes, hypertension, gynaecomastia, arthropathy, cardiomegaly, acanthosis nigricans

Bitemporal hemianopia and optic atrophy

Rare causes: MEN type 1, Mc-Cune Albright syndrome and Carney complex

212
Q

How does hypopituitarism / pituitary apoplexy generally present?

A

Sudden-onset headache behind eyes / around temples (90% of cases)

Associated nausea and vomiting

Sometimes meningism and reduced GCS

Bitemporal hemianopia visual field defect

May have 3rd nerve palsy

Usually have adrenal insufficiency (secondary) and other pituitary hormone deficiencies

213
Q

Questions to ask when suspecting pituitary apoplexy?

A

Gonadotrophin: no / irregular periods, reduced libido, hot flushes

GH: weight gain

TSH: weight gain, feeling cold, dry skin, hair loss, fatigue, constipation, puffy face

ACTH: dizzy on standing, tired, weak, nausea / vomiting, weight loss, abdo pain

PMH: previous pituitary surgery, radiation, cancer, head injury

NB you do not get salt wasting and hyperpigmentation in secondary adrenal insufficiency as you do in primary adrenal insufficiency

214
Q

Causes of hypopituitarism?

A

Pituitary tumour: often non-functioning macroadenoma

Iatrogenic: pituitary surgery / radiation

Compression / infiltration: meningioma, mets, sarcoid, abscess, craniopharyngioma

Pituitary apoplexy: acute infarction of a pituitary adenoma

Head injury / stroke / meningitis

Sheehan syndrome (infarction during blood loss in childbirth)

Empty sella syndrome

215
Q

Investigations to request for suspected hypopituitarism?

A

Bedside: obs, history, CBG

Bloods: FBC, UE, 9am cortisol (low), TFTs (low), LH / FSH (low), morning testosterone / oestradiol (low), IGF-1 (low), prolactin (low or raised)

Imaging: CT or MRI head and pituitary (MRI has >90% sensitivity for detecting pituitary apoplexy)

216
Q

Management of pituitary apoplexy?

A

Medical: hormone replacement

Surgical: transsphenoidal surgery if mass effect

217
Q

How to perform Weber’s and Rinne’s tests?

A

Rinne’s first: to assess conduction
- If normal, air > bone
- If sensorineural deafness, air > bone
- If conductive deafness, bone > air

If Rinne’s +ve, progress to Weber’s - to assess lateralisation
- If sensorineural, will localise to good ear.
- If conductive, will localise to affected ear.

218
Q

Ddx for psoriatic arthritis?

A

Rheumatoid arthritis

Other seronegative spondyloarthropathies: ank spond, reactive arthritis, enteropathic arthritis

Osteoarthritis

Crystal arthropathy (gout, pseudogout)

219
Q

What criteria are used to diagnose psoriatic arthritis?

A

CASPAR criteria - to diagnose PsA, need =>3 points:

  • psoriasis (current, past, FH)
  • psoriatic nail dystrophy
  • negative RF
  • dactylitis (current or past)
  • XR: juxta-articular new bone formation
220
Q

Name some subtypes of psoriatic arthritis.

A

Asymmetrical oligoarthritis: DIPJ + PIPJ, MCPJs + MTPJs, knee + hip, dactylitis

Predominantly DIPJ with nail changes

Athritis mutilans: telescoping of fingers

RA-like: wrist involvement

Axial only: asymmetrical sacroiliitis

221
Q

Extra-articular features of psoriatic arthritis?

A

Nail involvement in 80% - pitting, ridging, hyperkeratosis, onycholysis

Ocular involvement - conjunctivitis, iritis

222
Q

Investigations in suspected psoriatic arthritis?

A

Bedside: obs, history, urine dip, sexual hx (reactive arthritis differential)

Bloods: RF and anti-CCP, ANA, ESR / CRP, FBC, UE, LFT, HLA-B27 testing if sacroiliitis

Imaging: XR hands / feet: asymmetrical changes, no periarticular osteopenia, DIPJ involvement, erosion of terminal tufts, ‘pencil in cup’ deformity, sacroiliitis, juxta-articular new bone formation

223
Q

Management of psoriatic arthritis?

A

Lifestyle advice: weight loss, exercise, smoking cessation

NSAIDs

Steroid injections (but be aware this can cause skin to flare)

DMARD when NSAIDs fail, 3+ joints involved, PASI score =>10 (MTX, sulfasalazine, leflunomide)

Biologic: Anti-TNF / Ustekinumab / Apremilast

Surgical: joint replacement

Dermatology opinion on skin disease

224
Q

What two cell types are found in sarcoidosis?

A

Non-caseating granulomas and Langhans giant cells.

225
Q

Symptoms of sarcoid?

A

Eyes: dry eyes, blurred vision

Lymph nodes: enlarged

Lungs: dry cough, breathlessness, wheeze

Heart: chest pain, arrhythmias, heart palps

Hepatosplenomegaly

Joints: pain, arthritis, swelling

Skin: rashes, lupus pernio, erythema nodosum, skin lesions on back, subcutaneous nodules

Whole body: fatigue and weight loss

226
Q

Features of yellow nail syndrome?

A

Discoloured, dystrophic and thickened nails.

A condition associated with lymphoedema, pleural effusions and bronchiectasis in 40% of cases.

227
Q

What treatments do you know of for RA?

A

Bridging steroids for flares
Early DMARD’s

MTX +/- second agent, such as leflunomide, sulfasalazine

Anti-TNF (Infliximab)
Anti-CD20 (Rituximab)
T-cell co-receptor blocker (Abatacept)

228
Q

How do you quantify a RA flare?

A

DAS28

  1. Number of swollen/tender joints
  2. Global pain score
  3. ESR/CRP
229
Q

What are the diagnostic criteria for RA?

A

ACR/EULAR

  1. No. LARGE joints
  2. No. SMALL joints
  3. ESR/CRP
  4. Anti-RF/CCP
  5. > 6 weeks duration
230
Q

Characteristic hand signs of RA?

A

Wrist subluxation
Thickened synovium
Ulnar deviation
Boutoiniers (PIP flex.d)
Swan neck (PIP ex.d)
Z thumb (MP hyper-ext)
Rheumatoid nodules (elbows)
Arthroplasty scars

231
Q

What important extra-articular manifestations of Psoriatic arthritis are you aware of?

A

With all spondyloarthropathies:

  • aortitis
  • aortic regurgitation
  • iritis
  • apical lung fibrosis
  • colitis
  • enthesitis (specific for spondyloarthropathies)

DDx from other spondyloarthropathies by more asymmetry, and more peripheral joint involvement.

232
Q

What are the key differences between PsA and RA?

A

Equal sex distribution (RA more common in women)
Asymmetric
Distal (with some exceptions)
Enthesitis
Spondylitis in 40%!
Early bone deformity
Majority RF / CCP negative

233
Q

Which other condition can Mitral Stenosis mimic?

A

LVSD. If the mitral valve area is <1, this can mimic LV failure when the LV is functionally normal

234
Q

Clinical signs of HOCM?

A

Prominent a due to septal bulging
Double carotid impulse
Reverse split S2
MR (anterior motion of valve)
Louder during Valsalva (reduced pre-load = reduced Starling = obstructed outflow)

235
Q

Genetics of HOCM?

A

Autosomal Dominant

Several genes, all encoding sarcomeric proteins

Associated with Friedrich’s ataxia, Fabry’s, WPW

236
Q

What are the ECG findings of HOCM?

A

LVH criteria (S1+R6 >35mm)
Deep antero-lateral TWI
Deep infero-lateral Q waves

237
Q

Which medications must you avoid in HOCM?

A

Drugs that reduce pre-load as this accentuates the outflow gradient: nitrates, inotropes

238
Q

Management of HOCM?

A

Amiodarone if AF
Beta-blockers
Cardioverter defibrillator
Dual chamber pacemaker
Endocarditis prophylaxis

239
Q

What do you know about the genetics of Ehler Danlos syndrome (EDS)?

A

AD, AR, de novo mutations in gene encoding for collagen III

Principally affects joints, blood vessels, heart, eyes

240
Q

What are the complications of EDS?

A

Joints = deformities, pain, impaired function

Skin = easy bruising and fragility

Heart = MVP, MR, AR, Aortic dissection, cardiac conduction abnormalities

Vascular = haemorrhage

GI = bleeding, diverticulae, diaphragmatic herniation, colonic rupture

Eyes = glaucoma, retinal detachment

241
Q

What do you know about Osteogenesis imperfecta?

A

AD mutation usually in COL1A1/2 that codes for collagen I
This leads to
B: weak bones and atraumatic fractures
I: Eyes - blue sclera (loss of collagen = choroidal veins visible)
T: Teeth - dentinogenesis imperfecta
E: Ear: otosclerosis (conductive hearing loss)

Cardiac: Valvular disease - MVP, bicuspid aortic valve, AR/aortic root dilation

Rx:
1. Vit D/calcium/bisphosphonates
2. OT/PT
3. Hearing aids
4. Valve replacement/repair if needed

242
Q

DDX of blue sclera?

A

EDS

Marfan

Osteogenesis Imperfecta

Pseudoxanthoma elasticum

Diamond Blackfan (Pure red cell aplasia)

243
Q

What do you know about PXE?

A

AD/AR ABCC6

Abnormal MINERALISATION of ELASTIN

Mid-dermal deposition of calcium that triggers a metabolic response

Dermis: Pseudoxanthoma plaques and plucked chicken) in axilla/groin/neck

Eyes: Angoid streaks (85%)

Heart: AR, MR, MVP

Vessels: Early plaques, CAD, PVD, HTN, Renovascular disease

GI: haemorrhage due to friable vessels

244
Q

What are the consequences of Systemic Sclerosis?

A

GI: Oesophogeal dysfunction, dysphagia, malaborption, SIBO, PBC

Skin: Raynaud’s, gangrene

MSK: arthritis, osteopenia (malabsorption)

Renal: HTN, scleroderma renal crisis

Resp: ILD, pleural effusions

CVD: Restrictive cardiomyopathy, pericarditis, PHTN, conduction defects

245
Q

What is the usefulness of nailfold capilloroscopy in systemic sclerosis?

A

The nailfold is one of the few places you can visualise capillaries. There is significant capillary dropout due to microangiopathy, with regeneration of poorly constructed vessels in systemic sclerosis/Raynauds

246
Q

Diagnostic criteria for SLE?

A

ANA >1:80 and signs/symptoms/labs suggestive of lupus:

Labs: Anti-dsDNA, Anti-smith, Anti-cardiopilin, lupus anti-coagulant, low C3/C4

Constitutional
Neurological
Mucocutaneous
Photosensitivity
MSK
Serosal
Renal
Haematological

247
Q

What are the classic skin features of lupus?

A

Discoid lupus (sun-exposed)
Malar rash on face sparing naso-labial folds
Painless mucosal ulcers
Tender small joints

248
Q

What pregnancy complications of lupus do you know?

A

Auto-antibodies can cross the placenta and lead to congenital heart block (Anti-Ro)

249
Q

What are the renal manifestations of lupus?

A

6 different types of lupus nephritis, most common and most severe is diffuse proliferative lupus nephritis

250
Q

How do you treat SLE?

A

Mainstay is hydroxychloroquine

+/- other DMARDs like AZA, MMF, calcineurin inhibtors (ciclosporin/tac)

Steroids for flares

ACEi if proteinuria

251
Q

What are the systemic features of lupus?

A

Anaemia
Arthritis
Raynauds
Dermatomyositis
Discoid rash
Malar rash
Mouth ulcers
ILD, pleural effusions, pleurisy
pericarditis, Libman-Sacks endocarditis, PHTN
CNS involvement
Renal HTN, nephritis
Haematological - lymphadenopathy, splenomegaly
Eyes - Sjogren syndrome

252
Q

What are the non-spinal features of ankylosing spondylitis?

A

Anterior uveitis
Aortic regurgitation
AV conduction defects
Enthesitis
Apical fibrosis
Ank spond is a rare cause of amyloidosis

253
Q

Treatment of ankylosing spondylitis?

A

NSAIDs
Excercise
DMARD’s in severe cases
Biologics such as anti-TNF (adalimumab, etanercept) and anti-IL17
Rarely, surgery (spinal surgery is usually avoided)

254
Q

What are the signs of osteomalacia?

A

Bone fragility
Proximal muscle werakness
Muscle spasms (low Ca)
X ray - looser zones

Bloods:
Low Vit D, Low calcium, raised ALP (trying to stimulate bone actiivty). Low calcium stimulates PTH which stimulates phosphate excretion.
Low Vit D, Low Ca, Low Pho
Raised ALP and raised PTH

255
Q

What is osteoporosis and how is it different from osteomalacia?

A

Balance tips towards bone breakdown

Leads to reduced number of osteoid units, however the cells are functionally normal and there is preserved mineralisation (in contrast to osteomalacia where there is reduced mineralisation)

256
Q

What are risk factors for osteoporosis?

A

Post-meonopausal
Low calcium
Alcohol
Smoking
Steroids
Thyroxine
Senile osteoporosis
Reduced gravity/weight bearing excercises

257
Q

How do you assess disease severity of acute flares in UC?

A

Truelove and Witts’ criteria:

Mild: <4 stools a day

Mod: 4-6 stools a day, no sys upset, CRP <30

Severe: >6 and any signs of systemic upset

258
Q

What are the indications for surgery in a flare of IBD?

A

Refractory to medical therapy
Any evidence of dilatation on imaging
Any evidence of fistulating disease

259
Q

Are there concerns for malignancy in IBD?

A

Yes, if there is colitis there is concern for colorectal ca and PSC —> cholangiocarcinoma, particularly in UC

After 10 years - surveillance colonoscopy to detect early pre-malignant changes; further colonoscopy depending on risk level at that stage.

260
Q

How do you assess disease severity in psoriasis?

A

Psoriasis area and severity index (PASI), and body surface area involved

261
Q

What environmental triggers for psoriasis do you know?

A

Trauma
Sunburn
Stress
Smoking
Alcohol
Beta haemolytic strep
Drugs (Lithium, ACEi’s, beta-blockers, NSAID’s, terbinafine, anti-malarials)

262
Q

What variants of psoriasis do you know?

A

Plaque
Pustular
Guttate
Palmoplantar
Erythrodermic
Inverse psoriasis (flexor surfaces)

263
Q

What is Auspitz sign?

A

Stripping of skin in psoriasis reveals underlying hypertrophied dermal capillaries

264
Q

What genes are implicated in NF?

A

NF1 on chromosome 17
NF2 on chromosome 22
Both these genes are TSG’s. Their absence leads to the fibrous tumours arising from the nerves - fibromas

265
Q

What are the manifestations of NF2?

A

Schwannomas (acoustic neuromas)
Meningiomas
Cataracts
Retinal hamartomas
Less frequent skin manifestations

266
Q

Other than skin and nerve involvement, what other organs can be affected in NF?

A

Lungs - apical fibrosis
Heart - restrictive cardiomyopathy
Spine - scloliosis
Adrenal - phaeochromocytoma
Kidneys - Renal artery stenosis

267
Q

What are the responsible genes in Tuberous Sclerosis?

A

TSC1 and TSC2. Both are inherited in an autosomal dominant manner, and their absence leads to uncontrolled mTOR activity. This leads to benign tumours throughout the body (made up of various cell types) and an increased lifetime risk of cancer

268
Q

Which organs are most often affected in Tuberous Sclerosis?

A

Brain - gliomas, epindymomas, astrocytomas
Skin - angiofibromas (face), subungal fibromas, ash leaf spots, Shagreen patches
Kidneys - angiomyolipolas (can bleed)
Lungs - lymphangioleiomyomatosis (cysts)
Eyes - reintal hamartomas.

269
Q

What investigations need to be sent from an LP for suspected meningitis?

A
  • Cell count and differential
  • Protein, glucose, lactate
  • Gram stain and culture
  • Viral PCR

Bacterial meningitis
Raised WCC, predominantly Neutrophils
High protein
Low glucose
Positive gram strain

Viral meningitis
Raised WCC, predominantly lymphocytes
Normal/High protein
Normal glucose
Viral PCR+

270
Q

Complications of bacterial meningitis?

A

Severe morbidity
Deafness
Blindness
Cognitive impairment
Vascular - amputations due to sepsis

271
Q

What are the manifestations of hereditary haemorrhagic telangiectasia?

A

Skin - telangiectasia
Mucus membranes - epistaxis
GI - bleeding
Lungs - AV malformations
CNS - SAH, migraines, spinal AV = paraparesis
Liver - can lead to liver failure due to AV malformations

272
Q

What is your differential for telangiectasia?

A

Systemic Sclerosis
SLE
HHT
Mitral stenosis
Polycythaemia
Carcinoid syndrome
Ataxia telangiectasia

273
Q

OSA symptom scoring criteria

A

STOP-BANG questioniarre
Epworth (>11)

274
Q

OSA Diagnostic test?

A

Overnight polysomnography for apnoea hypopnea index

275
Q

What are triggers for dermatomyositis?

A

Genetic: HLADR3/DR5
Environment: Cocksackie virus, cancers

276
Q

Which cancers are associated with dermatomyositis?

A

An underlying malignancy is found in 20-25% of cases of dermatomyositis.
These are typically ovarian, lung, breast

277
Q

What are the clinical manifestations of dermatomyositis?

A

Skin:
Gottron’s papules
Heliotrope rash
Malar rash
Shawl sign
Hypertrophy of cuticles
Non-scarring alopecia.
Palmar hyperkeratosis (mechanics hands).
Photosensitive skin changes.

Muscles:
Prox weakness
Atrophy
Dysphagia
Resp muscle weakness

Lungs:
Fibrosing alveolitis
Organising pneumonia

278
Q

What investigations would you perform for suspected dermatomyositis?

A

FBC, UE
CK
ANA, Anti Jo1, Anti SRP, Anti M2
Muscle biopsy
Directed screening for underlying malignancy

279
Q

DDx for Raynaud’s

A

RA
SS
Lupus
Dermatomyositis
Polymyositis
Myeloproliferative disorders
Hep B/C
Thoracic outlet syndrome
Atherosclerosis
Carpel tunnel
Hypothyroidism
Drugs (beta blockers, cyclosporin)

280
Q

What are the differences between Raynaud’s disease (primary) and Raynaud’s phenomenon (secondary)?

A

Primary (R. disease) - <40y
No associated systemic features
NO nailfold capillaries
Never necrotic
Normal ESR
Normal ANA

281
Q

What drugs can cause Raynaud’s?

A

Beta blockers
OCP
Ciclosporin
Chemo

282
Q

Treatment for Raynaud’s

A

CCB’s
Prostacyclins
Nitrates

283
Q

Categorise lymphomas

A

Commonly classified as Hodgkin’s (Reed Sternberg B-cells) and non-Hodgkins.

Non-Hodgkins includes:
High-grade
Lymphoblastic (behaves like ALL)
B-cell (Burkitt’s, DLBCL, Mantle cell, Follicular)

284
Q

How do you stage Hodgkin’s lymphoma?

A

Ann-Arbour staging
1. Single node
2. More than one node, same side of diaphragm
3. Above and below diaphragm
4. Extra-nodal

B symptoms at any stage

285
Q

What is the main treatment for high grade NHL?

A

R-CHOP
Rituximab
Cyclophosphamide
Hydroxydaunorubicin
Oncovincristine
Prednisolone

and then maintenance Rituximab

286
Q

What is the treatment for Hodgkin’s lypmphoma?

A

ABVD
Adriamycin
Bleomycin
Vincristine
Doxarubicin

287
Q

What are the cardinal features of anti-phospholipid syndrome?

A

Thrombocytopenia (antibodies against platelets)
Recurrent miscarriages
Arterial (stroke) and/or venous clots (DVTs)
Livedo reticularis

288
Q

What tests would you perform in suspected antiphospholipid syndrome?

A

Autoimmune screen
anti-cardiolipin
Anti-beta2glycoprotein 1
anti-phospholipid
Lupus anticoagulant

289
Q

How do you treat antiphospholipid syndrome?

A

Primary thromboprophylaxis - low dose aspirin
Secondary thromboprophylaxis - warfarin (INR 2-3, increased to 3-4 if clot on warfarin)

290
Q

What causes for strokes in young patients do you know off?

A

Extra-cranial - neck hyperextension
Vasculitis
Cardiac - embolic (mxyoma, endocarditis, valvular pathology, AF)
Substance misuse - cocaine, methamphetamine
Antiphospholipid syndrome
Familial hyperlipidaemia
Sickle cell disease
Rare metabolic/mitochondrial causes such as MELAS

291
Q

Causes for erythema nodosum?

A

STREP THROAT!
TB
Sarcoid (Lofgren’s - classical triad of fever, hilar lymphadenopathy and EN)
Myeloproliferative disorders
IBD
Behcet’s
Drugs (OCP, penicillin, sulphonylureas)

292
Q

What causes erythema multiforme?

A

HSV
CMV, EBV, influenza, COVID-19
Medications - OCP, penicllin, carbamazapine
Vaccinations
SLE
Sarcoidosis
Malignancy

Mycoplasma pneumonia - now considered a distinct entity

293
Q

Name some systemic causes for livedo reticularis

A

Antiphospholipid syndrome
Polycythaemia rubra vera
SLE
Polyarteritis nodosa
GPA/EGPA
Walendenstrom’s
Cholesterol embolism

294
Q

What do you know about bullous pemphigoid?

A

Bullous = Basement

Autoimmune type 2 hypersensitivity reaction against hemidesmosomes that anchor basal cells to basal membrane

IgG targets BPAG1+2. This leads to mast cell degranulation and inflammation

Genetic component and environmental triggers (furosemide, penicillamine, NSAIDs, abx)

Nikolsky sign negative (no intraepidermal splitting)

Rx topical and sytemic steroids

295
Q

What do you know about pemphigus vulgaris?

A

Autoimmune type 2 hypersensitivity reaction against intra-epidermal desmosomes that hold keratinocytes together in the stratum spinosum layer

Can either affect mucosal cells only (Desmoglein 3) or skin+mucosal cells (Desmoglein 1+3)

Histologically there is tombstoning (intra-epidermal disruption of keratinocytes)

Nikolsy sign positive (lateral pressure causes a split between upper and lower layers of epidermis)

Rx: topical and oral steroids, rituximab, oral rinses with lidocaine

296
Q

What do you know about Peutz Jegher’s syndrome?

A

Autosomal dominant mutation in the tumour suppressor gene STK11 that is expressed throughout the GI tract and also various other tissue including skin, breast, lung, ovaries, testicles

Leads to the development of hamartomas in the GI tract that can become malignant. These can lead to bleeding and obstruction.

Skin manifestations include flat mucocutaneous melanocytic macules on the mucosal surfaces and also palms and soles

Risk of GI, breast, lung, ovarian, testicular cancer

297
Q

What is necrobiosis lipoidica?

A

Typically affects young female diabetic patients
It is also associated with hypertension, thyroid disease and obesity
It is a disorder of BLOOD VESSELS
Starts as a red papule, becomes a plaque, and then slowly enlarges to a yellow patch with a red rim
Can ulcerate and turn into a squamous cell cancer

298
Q

What is pyoderma gangrenosum?

A

Extremely painful
Rapidly enlarging tender full-thickness ulcer with blue borders
Non-infectious neutrophilic dermatosis
More common in over 50’s
Frequently associated with autoimmune conditions such as IBD, RA, GPA

Rx: Urgent immunosupression

299
Q

Talk me through the Wells DVT score

A

Paralysis
Cancer
Immobilised
Prev DVT

Is an alternative diagnosis as likely?
Calf swelling, leg swelling, collaterals, tenderness along deep venous system, pitting oedema

1 = D-Dimer
>=2 Doppler

300
Q

Talk me through the Wells PE score

A

DVT
Cancer
Immobilised
Prev DVT

Is PE as equally likely as other diagnosis

HR
Haemoptysis

1 = D-Dimer
>2 = CTPA/VQ

301
Q

What are the components of a thrombophilia screen?

A

Factor V Leiden
Prothrombin mutation
Protein C
Protein S
Anti-thrombin III (rare but aggressive)
Drug history (OCP)

302
Q

Are there any scoring criteria you can use to risk stratify patients for inpatient/outpatient Rx for PE?

A

PESI Score predicts 30-day mortality and takes into consideration history, and clinical findings

303
Q

What are contraindications for thrombolysis of PE?

A

Intracranial disease/bleed
Ischaemic stroke (<3mo)
Brain/spine surgery (<3mo)
Internal bleeding
Coag disorder
Recent head trauma

304
Q

VQ vs. CTPA risks to pregnant pt?

A

Both have radiation - CTPA radiation comes from the X-rays, VQ radiation comes from the dye.
CTPA = higher dose of X-ray radiation to breast tissue
VQ = higher dose of dye - ionising radiation to fetus, but still very small amounts

305
Q

Do you know the inheritence of Retinitis Pigmentosa?

A

AD, AR, X-linked recessive, and 30% are de novo.

306
Q

Do you know of any syndromes involving retinitis pigmentosa?

A

Kearns-Sayre: Mitochondrial. Ophthalmoplegia + cerebellar ataxia.

Usher: AR. Sensosineuronal deafness.

Refsum: Ataxia + muscle wasting.

Alport: X linked recessive. Type IV collagen. Haematuria + GS + SN deafness.

307
Q

What is the treatment of proliferative diabetic retinopathy?

A

If disease involves fovea = anti-VEGF (benralizumab)
If disease does not involve fovea = laser photocoagulation

308
Q

What are the clinical signs and symptoms of a retinal vein occlusion?

A

Reduced visual acuity
RAPD
SWOLLEN disc
Retinal haemorrhages (stormy sunset)
Might develop acute angle closure glaucoma

309
Q

What are the causes of a retinal arterial occlusion?

A

The retinal artery is a terminal branch of the ophthalmic artery. Its occlusion can produce retinal infarction.

Can be:
1. Embolic
2. Rapid raised IOP
3. GCA
4. Profound hypotension
5. Vasospasm
6. Hypercoagulable states

310
Q

What are the signs and symptoms of retinal artery occlusion?

A

Sudden painless visual loss
Pale retina
Cherry red spot on fovea (choroidal arteries unaffected)
RAPD

311
Q

What is the treatment for central retinal artery occlusion?

A

Occular massage
IV acetazolamide/anterior segment paracentesis (reduce IOP)
Treat underlying factor

312
Q

What are the clinical signs of macular degnereation?

A

Dry: Drusen + atrophy
Wet: Neovascularisation in choroid

313
Q

What are the symptoms of macular degeneration?

A

Central visual loss
Distortion

314
Q

What is the treatment for macular degeneration?

A

Anti-VEGF (ranibizumab)

315
Q

What are the clinical findings of anterior uveitis?

A

Red
Painful
Asymetric pupil

316
Q

What conditions is uveitis associated with?

A

Ank Spond
Behcet’s
MS
Infections: HSV, TB, HIV

317
Q

What are the risk factors for acute angle closure glaucoma?

A

Asian, short axial eyeball, older age, pupillary dilation

318
Q

What are the symptoms of acute angle closure glaucoma?

A

Extreme pain
Blurred vision
Haloes
Red eye
Mid-dilated or fixed dilated pupil
Corneal oedema
Raised IOP

319
Q

What is the treatment for acute angle closure glaucoma?

A

Topical betablockers/pilocarpine
IV acetazolamide
Peripheral laser iridotomy (hole in iris)

320
Q

What is keratitis?

A

Inflammation of the cornea
Sight-threatening emergency
Caused by infection (bacterial, fungal, viral) or inflammation (blepharitis induced irritation)

Ix - slit lamp and fluorescein = corneal ulcer

321
Q

Scleritis vs. episcleritis?

A

Both associated with inflammatory conditions such as IBD and RA
Scleritis is painful, episcleritis is not

Both look pretty similar

Scleritis is associated with photophobia and gradual decrease in visual acuity

In episcleritis, injected vessels are mobile with gentle pressure and will blanch with phenylephrine drops

Scleritis needs oral NSAID’s /steroids, most cases of episcleritis can be managed conservatively

322
Q

What do you know about deQuervain’s thyroiditis?

A

Typically post a viral illness

Phase 1: Lymphocytic infiltration: Hyperthyroidism, raised ESR

Phase 2: euthyroid

Phase 3: Hypothyroidism for weeks-months

Phase 4: Back to normal

Ix: Thyroid scintigraphy: globally reduced uptake of I-131

Rx:
Self-limiting
Pain control - NSAIDs
Steroids if hypothyroid

323
Q

What classes of diabetic drugs do you know for patients with T2DM?

A

Biguanide: increase liver and muscle insulin sensitivity (metformin)

SGL-2 inhibitor: Inhibits re-absorption of glucose (dapaglifozin).

DPP4 inhibitors: reduce breakdown of incretins (sitagliptin, linagliptin). Caution if hx pancreatitis/pancreatic cancer. Can use linagliptin in renal impairment

Sulphonylureas: Increase insulin secretion (glizlazide) - risk of hypo’s

Pioglitazone: PPAR-gamma activator = more glucose transports - risk of fluid retention, osteoporotic fractures, and bladder cancer

324
Q

What are the complications of spina bifida?

A

Urinary stasis + recurrent infections
Lower limb spasticity
Hydrocephalus
Arnold-Chiari malformations
Recurrent meningitis
Syrinx

325
Q

What conditions are associated with acanthosis nigricans?

A

T2DM
Obesity
PCOS
Acromegaly
Cushing’s

326
Q

What are the features of Williams syndrome?

A

Spotaneous microdeletion
Mild intellectual impairment
Characteristic facies with broad forehead
Supravalvular aortic stenosis

327
Q

Steroids in ascending order of potency

A

Hydrocortisone (Mild)
Betamethasone 0.025%
Fluticasone/Betamethasone 0.1%
Clobetasol

328
Q

What is the pathophysiology of eczema?

A

Allergen enters the dermis and triggers a Type I hypersensitivity reaction. On repeat exposure there is IgE mediated mast cell degranulation.

Self-perpetuating cycle of skin breakdown, skin dryness, and itching.

329
Q

Causes of galactorrhoea?

A

Physiological (pregnancy)
Prolactinoma
Exercise, stress
Acromegaly (1/3 of patients)
PCOS
Hypothyroidism (Raised TRH stimulates prolactin release)
Drugs (metoclopromide, domperidone, haloperidol, rarely SSRI’s)

330
Q

Tell me about sickle cell anaemia

A

AR

HBB gene (beta globin)

Low haptoglobin (intravasc haemolysis)
Scleral icterus, jaundice, bilirubin gallstones

Bone marrow hyperplasia - enlarged skull

Hepatomegaly (extramedullary haematopoiesis)

Autosplenectomy (R/O Strep, haemophilius, Neisseria, Salmonela)

Chest syndrome

Renal necrosis

Priapism

Rx: Prophylaxis, treat infections early, pain relief, hydroxyurea (increases gamma globin = ^ HbF)

331
Q

Management of infective endocarditis?

A
  1. A-E approach, stabilise pt. Blood cultures before treating if possible. Sepsis six, 3x sets of BCs from peripheral sites at different times.
  2. IV abx: benzylpenicillin + gentamicin OR vancomycin if pen-allergic. If HACEK organism suspected, treat with ceftriaxone OR ciprofloxacin. 4-6 weeks of IV abx.
  3. Surgery: indications include heart failure, cardiogenic shock, aortic root abscess, expanding vegetation, AV block, prevention of septic emboli
  4. Temp cardiac pacing is recommended if the pt is in AV block secondary to aortic root abscess.
  5. Refer to endocarditis MDT
332
Q

Key points in an alopecia history?

A

Timing: when did it start, getting worse, sudden/gradual, anything made it better or worse?
Patchy or widespread?
Scarring: more likely to be autoimmune-related
Red/scaly
Scalp/beard/body hair/eyebrows/eyelashes affected?
Does it re-grow white?
Nails

PMH: AI (DM, vitiligo, coeliac, thyroid), tiredness
FH: hair loss?
DH: allergies, chemo, COCP, heparin, warfarin
SH: diet, stress, illness, trauma, surgery, pregnancy

333
Q

Examination features in alopecia history?

A

Hair: patchy/widespread, inflamed, scarring, exclamation mark hairs, white hair
Beard, brows, eyelashes, body hair
Nails
Don’t miss vitiligo!
Examine thyroid
Check for conjunctival pallor

334
Q

Management of alopecia areata?

A

Mild: reassurance
Topical potent corticosteroid / intralesional triaminoclone
Dithranol or minoxidil 5%
If severe: topical immunotherapy, systemic steroid, PUVA

Trigger control
Support groups, wigs

335
Q

Types of non-scarring and scarring alopecia?

A

Non-scarring
Androgenic/ageing
Telogen effluvium
Drug-induced
Areata/totalis/universalis

Scarring
Trauma
SLE
Lichen planus
Scleroderma morphoea

336
Q

Investigations to perform in a case of alopecia?

A

Bedside: fungal culture (scraping)

Bloods: TFTs, FBC, iron/ferritin, glucose, ANA (lupus), syphilis serology

Special: dermoscopy (exclamation-mark hairs vs cadaverised hairs), skin biopsy if diagnosis unclear

337
Q

Ddx of an acute joint?

A

Haemarthrosis

Trauma

Septic arthritis

Crystal arthritis (gout, pseudogout)

Reactive arthritis

Monoarticular presentation of RA / psoriatic / enteropathic arthritis

Bursitis / cellulitis / OM

Avascular necrosis

338
Q

Investigations in suspected acute joint?

A

Bedside: urine dip + culture, stool culture, STI screen, ECG

Bloods: FBC, UE, LFT, CRP, ESR, Ca, Mg, ferritin, TFT, urate, Hba1c, lipids, blood cultures
(Consider: RF, anti-CCP, ANA, HLA-B27 if >1 joint involved)

Imaging: XR joint, MR joint if OM suspected

Special: aspirate joint (pre-abx)
(Contraindicated in prosthetic joints, cellulitis, overlying psoriasis, high INR, low plts)

339
Q

Tell me what you know about septic arthritis

A

Common organisms: staph aureus, group A/B strep, gram neg bacilli, gonococcal

Blood culture +ve in 50%, aspirate is +ve in 70-90%

Cause: haematogenous spread, adjacent OM, soft tissue infection near joint, iatrogenic (arthroscopy), trauma

340
Q

Tell me what you know about gout.

A

Crystal deposition disease

Disorder of purine metabolism, resulting in hyperuricaemia

Typically monoarthritis of 1st MTP joint, can be polyarticular

Males > females

Can become chronic, presence of tophi indicates severe, recurrent gout. Can get uric acid renal stones and nephropathy

341
Q

Gout management?

A

Lifestyle changes: reduce red meat / purine intake, alcohol, weight loss, exercise

Acute phase:
- rest / ice joint, stop diuretics, address lifestyle factors
- NSAIDs + PPI (diclofenac) for 1-2 weeks
- Colchicine 2nd line
- Steroids 3rd line

Prophylaxis:
- Allopurinol (not during acute attack, delays resolution)
- Offer if 2 attacks in 1y / tophi / erosions / renal insufficiency / renal stones / diuretics / pre-chemo
- Aim for urate <300 micromol/L
- Monitor U+E and titrate according to urate
- Start 2wks after attack settled
- Alternative: febuxostat

342
Q

Management of reactive arthritis?

A

Bed rest, ice, splints
Treat infection / contact tracing
NSAIDs
IA steroid / oral steroid
DMARDs: MTX, sulfasalazine
Most recover within 1y, 20% become chronic. Complications more likely if HLA-B27 positive.

343
Q

Investigations for suspected pseudohypoparathyroidism?

A

Bedside: ECG (QT prolongation, heart block, VF), 24h urinary calcium

Bloods: calcium, magnesium, potassium, phos, pH (VBG), PTH, vit D, UE, ALP, amylase, TFTs, ACTH, adrenal abs, genetic testing

Imaging: renal USS (stones), brain MR (basal ganglia calcification), XR hands (shortened metacarpals)

344
Q

Tell me about pseudohypoparathyroidism

A

Kidney and bone resistance to PTH: low calcium, high phos, high PTH, low calcitriol, normal UEs.

No urinary cAMP response to PTH.

Type 1a: GNAS1 mutation. Albright Hereditary Osteodystrophy (AHO) - round face, short stature, short 4th and 5th metacarpal bones, obesity, subcutaneous calcifications and developmental delay.

Type 1b: No phenotypic features of AHO, but similar biochemistry.

Type 2: No phenotypic features of AHO. Normal / raised urinary cAMP response.

345
Q

Management of pseudohypoparathyroidism?

A

Management of hypocalcaemia and refer to endocrinology.

May require screening for other endocrinopathies.

346
Q

What are some causes of hypocalcaemia?

A

Hypoparathyroidism (low PTH, low calcium)

PTH resistance (pseudohypoparathyroidism)

CKD

Vitamin D deficiency

Bisphosphonates, calcitonin, phenytoin

Acute pancreatitis

Blood transfusion

Rhabdomyolysis

Tumour lysis syndrome

Hyperphosphataemia

347
Q

How would you localise a lesion in a suspected stroke between the MCA and PCA?

A

Macular sparing is usually seen in PCA lesions due to the supply from the MCA.

If there is no macular sparing within the HH, then you can localise the lesion to the MCA.

The weakness/numbness would be expected on the contralateral side to the brain lesion.

348
Q

What are some of the complications that can occur in patients after acute management of ischaemic stroke?

A
  • Thromboembolic disease - give IPC boots
  • Risk of further strokes
  • Risk of haemorrhagic transformation of ischaemic strokes
  • In acute and long-term, there is a risk of seizures
  • Swallowing difficulties - risk of aspiration pneumonia
  • Higher risk of falls
  • Urinary and faecal incontinence
349
Q

What are the important aspects of secondary prevention in stroke?

A

Dual antiplatelet therapy (aspirin + clopi)

Lifestyle mods: BP, diet, statins, reduce salt intake, stop smoking, reduce alcohol intake

Further risk factors such as diabetes should be addressed using current guidelines

350
Q

What is the treatment and prognosis for a reactive arthritis?

A

Treatment includes:
- Treating the underlying cause (diarrhoeal illness, gonococcal / chlamydial infection)
- Supportive: analgesia, re-hydration if diarrhoeal illness, abx if sexually-transmitted infection

351
Q

What investigations would you suggest for a suspected reactive arthritis?

A

Bedside: stool culture, obs, sexual and travel hx.

Bloods: UE, LFTs, CRP/ESR, FBC, cultures, RF + anti-CCP (in case it is first presentation of RA instead).

Imaging: XR of the joint.

Special: aspirate joint, with culture and microscopy looking for crystal arthropathy or septic arthritis.

352
Q

Can you think of any genetic links to reactive arthritis?

A

Reactive arthritis is associated with the HLA-B27 gene.

This gene is widely associated with the seronegative spondyloarthropathies which include enteropathic arthritis, reactive arthritis, ankylosing spondylitis and psoriatic arthritis.

353
Q

What features on ECG would indicate right heart strain in the context of a suspected PE?

A

‘S1 Q3 T3’

A large S wave in lead I
Q wave in lead III
Inverted T waves in lead III

354
Q

How would you investigate for suspected PE?

A

Bedside: observations, ECG (looking for signs of right heart strain)

Bloods: FBC, UE, coag profile, CRP (infection), LFTs

Imaging: CTPA (to exclude PE), CXR (exclude pneumonia or PTX)

Calculate Wells’ score:
- If patient <2 points - low risk, do d-dimer
- 2-5 points - consider d-dimer or CTPA
- >=6 points - do CTPA

Alternative rule-out criteria = PERC score (8-item clinical criteria who identify which pts are low risk of PE and can be discharged from ED).

355
Q

How would you manage a patient with PE?

A

Initially start with treatment dose LMWH.
Then switch to oral anticoagulation: warfarin or NOAC.

Treatment duration
If provoked PE: 3 months
If unprovoked PE: 6 months

If acutely unwell / haemodynamically unstable, consider thrombolysis with alteplase, or surgical thrombectomy.

356
Q

How do you assess hyponatraemia?

A

Need to assess volume status first.
If hypovolaemic - is urinary Na >20 mmol/l?
- Yes: renal Na loss
- No: loss elsewhere

If euvolaemic - is urine osmolality >300mOsm/kg?
- Yes: SIADH
- No: water intoxication, severe hypothyroidism, Addison’s disease

If hypervolaemic: nephrotic syndrome, CCF, cirrhosis, renal failure

357
Q

Management of hyponatraemia?

A

Fundamentals: treat underlying cause + restore normal volume status.

Acute severe hyponatraemia: IV hypertonic saline bolus (100ml 3% NaCl) cautiously.

Hyponatraemia without neurological symptoms:
- correct by no more than 6mmol/L in first 6h
- then no more than 10mmol/L in first 24h
- management depends on cause + fluid status

Hypovolaemic: rehydration with 0.9% normal saline
Euvolaemic: fluid restrict 1.5L per 24h
Hypervolaemic: fluid restrict 1.5L per 24h

(All with regular monitoring of serum sodium levels - if done too fast can cause osmotic demyelination syndrome).

358
Q

List some causes of pseudohyopnatraemia.

A

Hyperlipidaemia
Hyperglycaemia
Hyperproteinaemia
Hyperbilirubinaemia

359
Q

What investigations would you like to do for suspected hyponatraemia?

A

Bedside:
- urine sodium (differentiates between renal vs extrarenal causes of hypoNa) - very high sodium in renal causes and SIADH
- urine osmolality (to confirm SIADH - increased urine osm + low serum osm suggests SIADH)

Bloods:
- UEs, serum osm (decrease in serum osm to be expected. If normal, suspect pseudohyponatraemia)
- LFTs (albumin), lipids (if pseudo suspected), glucose
- Cortisol (exclude Addison’s), TFTs (exclude hypothyroidism)

360
Q

Causes of erythema nodosum?

‘SORE SHINS’

A

Streptococcal infections
OCP
Rickettsia
Eponymous (Behcet’s)
Sulphonamides
Hansen’s disease (leprosy)
IBD / Idiopathic
Non-Hodgkin’s lymphoma
Sarcoidosis

361
Q

Treatment of idiopathic intracranial hypertension?

A

Medical:
- Acetazolamide
- Mannitol

Surgical:
- Optic nerve sheath fenestration
- CSF shunting (VP shunt)

Other:
- Weight loss

362
Q

Causes of hypertension?

A

Essential (94% - assoc with age / obesity / salt / alcohol)
Renal - CKD
Endocrine - Conn’s, Cushing’s, Acromegaly, Phaeochromocytoma
Aortic coarctation
Pre-eclampsia (pregnancy)

363
Q

Investigations for diagnosis of ank spond?

A
  • Mainly clinical history + exam, with supporting radiological evidence
  • Young patients (<40y) with positive FH
  • Plain spine XRs: erosions / sclerosis of SI joints, with squaring of vertebrae
  • Bloods: raised CRP / ESR, anaemia of chronic disease
  • Genetic screening: HLA-B27
  • CXR / HRCT: if suspecting lung fibrosis, consider PFTs